CHA 1 Quizzes

अब Quizwiz के साथ अपने होमवर्क और परीक्षाओं को एस करें!

A nurse has an order to give 15 mg IV Ketorodol (Torodol). The dose available is 30 mg/4 mL. How much will the nurse draw up?

2mL

a patient with cystic fibrosis is newly prescribed pancrelipase. The prescriber has ordered 500 lipase units/ kg/ meal. The patient weighs 185 pounds. How many units of lipase will the patient take per meal? (Round all math at the end of the equation. Enter the number to the nearest unit and do not include a label or comma. Example: 5000)

42045 (rational: 185/2.2= 84.09lbs x 500 units)

The nurse is caring for a 6-year-old child who had abdominal surgery for a ruptured appendix. When performing a pain assessment, the nurse identifies that the child's chin is quivering and the legs are restless. The child is squirming in the bed and whimpering softly. When the nurse hugs the child and offers reassurance, the child's whimpering subsides. What is the child's pain assessment score using the FLACC scale?

6

10-year-old child is hospitalized following an electrical burn. Which assessment findings should the nurse use to determine adequate fluid balance? Select all that apply A. Urine output 2ml/kg/hr B. Monitoring blood pressure C. Monitoring fluid intake D. Comparing the radial pulse with the apical pulse E. Assess for irritability and headaches

A, B, C, E (Rational: A child with an electrical burn should be closely monitored for fluid balance. Due to increased capillary permeability, large proteins can "leak" into the interstitial space, pulling fluid with them. This leaves the patient with a "dry" vascular space as well as edema. The nurse should monitor for signs of fluid volume deficit, especially related to the perfusion for the central organs (brain, heart, lungs, kidneys). These would include signs of hypotension, low urine output and headaches and irritability. The nurse should also closely monitor I&O and encourage adequate fluid intake)

The nurse educator is developing a presentation for the medical unit staff on pain in geriatric patients. Which statement should the nurse include in the presentation? a. "Patients with cognitive delays, such as Alzheimer's or dementia, should be assessed using a behavior-based pain scale." B. "The nerve endings have aged in geriatric patients and they do not have as much pain." C. "Patients over 80 years old should not use NSAIDs for pain relief." D. "Patients over 80 years old are opioid naive and are likely to experience respiratory depression when prescribed opioids."

A.

The nurse is caring for a patient with lung cancer that is experiencing a side effect from chemotherapy treatment. The nurse realizes that which side effect is most concerning? a. Diarrhea b. Nausea c. Weight loss d. Alopecia

A. (rational: diarrhea can lead to fluid and electrolyte imbalance which is a circulatory issue. Nausea and weight loss are common side effects of chemotherapy and loss of hair is more of a psychosocial concern.)

The nurse is caring for a patient with a calcium level of 7.1. Which complaint indicates that the patient is experiencing a complication related to this lab value? A. Prolonged bleeding when removing an IV B. Development of a DVT C. Muscle pain and weakness D. Headaches and dizziness

A. (Rational: A calcium level of 7.1 is low and the patient should be monitored for signs of hypocalcemia which can cause hyperexcitable symptoms. Option A is correct because calcium is a cofactor at every step-in blood-clotting cascade. When hypocalcemia is present, blood takes longer to clot. Clients should be taught to avoid activities that can result in injury to prevent excessive bleeding or bruising and healthcare workers should monitor for bleeding in the skin, stool, and urine. Option B is incorrect because low calcium causes the blood to clot slower, where hypercalcemia causes hypercoagulability and puts the patient at risk for developing blood clots. Option C is incorrect because muscle pain and weakness are symptoms of hypercalcemia. Option D describes symptoms related to change in fluid and sodium values)

The nurse is assigned 4 patients on a medical unit. Which client is at greatest risk for developing hyponatremia? A. 35-year-old client is receiving D5W while waiting abdominal surgery B. 45-year-old client who is taking a sulfonamide antibiotic for a UTI C. 55-year-old client who is receiving IV ketorolac following a hip replacement D. 65-year-old client who is taking digoxin for a. fib

A. (Rational: D5W contains 5% dextrose solution and no sodium or any other electrolyte. Normal sodium excretion paired with this hypotonic solution can lead to sodium loss and water gain. The patient in option B would not have sodium levels affected by a UTI or sulfonamide antibiotic. The patient in option C would not be at risk for hyponatremia as digoxin does not affect sodium level. Digoxin levels can be affected by potassium levels)

A patient is receiving 2g of magnesium sulfate IV to correct a serum magnesium level of 1.4. Which of the following assessments would alert the nurse to immediately stop the infusion? A. Absent patellar reflex B. Two episodes of diarrhea C. Premature ventricular complexes on telemetry D. Blood pressure increased from 120/60 to 135/80

A. (Rational: Magnesium maintains normal nerve and muscular function. With too much magnesium the neuromuscular signal is blocked causing muscle weakness, delayed or absent deep tendon reflexes, and slurred speech. A higher toxicity levels magnesium serves as a calcium channel blocker causing bradycardia, hypotension, not hypertension, arrythmias with a prolonged P-R and widened QRS interval. Option B is incorrect because with magnesium replacement diarrhea is an expected side effect and would not warrant stopping the infusion. Option C is incorrect because PVCs are not a side effect of magnesium. Option D, an increase in blood pressue, is incorrect as elevated magnesium levels will cause hypotension rather than hypertension.)

The nurse is teaching a student about using IV fluids to manage fluid imbalances. Which statement by the nurse is correct regarding a colloid solution? A. "Colloid solutions draw fluid from the interstitial space to the intravascular space." B. "Colloid solutions distribute volume equally" C. "Colloid solutions draw fluid from the intracellular space to the intravascular space" D. "Colloid solutions are the same as crystalloid solutions"

A. (Rational: colloid solutions draw fluid from the interstitial space to the intravascular space. They are known to be volume expanders because their molecules are too large to pass through semi-permeable membranes meaning they remain in the intravascular compartment. One of the most common colloids used is albumin. Colloids are indicated for patients exhibitying hypoproteinemia, and malnurished states, as well as for those who require plasma volume expansion by who can not tolerate large infusions for fluid. Patients undergoing orthopedic surgery or reconstructive procedures with an elevated potential for thrombus formation may also benefit from colloid solutions.)

Prior to administering a patients scheduled dose of enoxaparin (Lovenox), which lab value should the nurse evaluate? a. Platelets b. INR c. aPTT d. PT

A. (Rationale- INR and PT are associated with warfarin (coumadin) and interfere with clotting factors II, VII, IX and X. aPTT is associated with Heparin and inhibits thrombin or factor IIa. aPTT is routinely drawn based on the organizations protocol with administration of IV heparin. Other indications may include bleeding disorders and or pre-operative studies. Enoxaparin is known as low molecular weight heparin (LMWH) and platelets are evaluated as needed to prevent thrombocytopenia. LMWH does not technically require routine lab draws.)

The nurse is completing discharge instructions for a patient newly started on rosuvastatin (Crestor) for a serum cholesterol level of 275 mg/dL. Which of the following statements made by the patient indicates teaching has been effective? a. "I should avoid taking this medication with grapefruit juice." b. "I should take this medication with food." c. "If I skip a dose, I can double up the next day." d. "This medication will lower my high-density lipoproteins."

A. (Rationale- Rousuvastatin (Crestor) is used together with diet to treat hyperlipidemia. It works to lower the blood levels of LDL cholesterol to increase levels of HDL cholesterol and lower triglycerides. Grapefruit juice can potentiate drug effects. Med can be taken with or without food. Medications should never be doubled up.)

four patients arrived to the ED with vascular issues. Which patient should the nurse see first? A. the patient with a history of atherosclerosis who has severe back pain and is hypotensive B. a patient with chronic venous insufficiency and a temperature of 100.6 C. a patient with a 40-year cigarette pack history with complaints of foot numbness D. a patient with chronic venous insufficiency complaining of calf tenderness

A. (Rationale- The patient with a history of atherosclerosis, severe back pain and is hypotensive could indicate an aortic aneurysm (AAA). This is a medical emergency and warrants immediate evaluation.)

the nurse is caring for a patient suspected of heart failure. which finding in the patient's medical history is a risk factor for heart failure? A. aortic stenosis B. acute pancreatitis one month ago C. chronic fatigue syndrome D. pleural effusion

A. (Rationale: Aortic Stenosis is the narrowing of the valve in the large blood vessel branching off the aorta. This narrowing keeps the valve from opening fully, reducing blood flow to the body and making the heart work harder. This can cause the left ventricle to thicken and enlarge. Eventually the extra work of the heart can weaken the left ventricle and lead to heart failure.)

A patient with a myocardial infarction (MI) has been treated with thrombolytic therapy. Which intervention should the nurse implement to prevent embolic complications? A. Administer SQ heparin B. Apply ice to the injection site C. Place the patient in Trendelenburg position D. Instruct the patient to take slow deep breaths

A. (Rationale: Following clot lysis, large amounts of thrombin are released, increasing the risk of vessel reocclusion. To maintain vessel patency, IV or low-molecular weight heparin and aspirin are prescribed.)

which response occurs when a patient with heart failure experiences a decrease in cardiac output? A. activation of sympathetic nervous system B. activation of parasympathetic nervous system C. inactivation of the renin angiotensin system D. suppression of the cytokine release

A. (Rationale: Heart failure can cause reduced cardiac output. The neurohormonal responses that occur are activation of the sympathetic nervous system and the renin-angiotensin-aldosterone system (RAAS). This results in vasoconstriction. The body will attempt to compensate for the low cardiac output by increasing blood pressure, holding on to sodium and water and increasing the heart rate.)

the nurse is preparing to administer furosemide (Lasix) and carvedilol (Coreg) to a patient with heart failure. the patient has 2+ pitting edema in the legs, a previous shift urine output of 400mL, a blood pressure of 134/74 a heart rate of 78. which action should the nurse take? A. administer the furosemide and the carvedilol B. administer the furosemide but hold the carvedilol C. administer the carvedilol but hold the furosemide D. hold the furosemide and the carvedilol

A. (Rationale: Lasix decreases blood pressure and is potassium wasting. Carvedilol is a noncardioselective beta blocker and decreases the heart rate and blood pressure.)

A nurse is discharging a 70-year-old patient home with chronic pain. The patient has a history of congestive heart failure. Which pain medication should the nurse clarify with the health care provider? a. Ibuprofen b. Oxycontin c. Codeine d. Acetaminophen

A. (Rationale: NSAIDs can cause sodium and water retention and should be used carefully in patients with CHF.)

A child with an asthma attack is treated in the ED with epinephrine. Despite receiving the medication, the child is still agitated, sweating profusely, has a RR of 30 breaths/minute and an oxygen saturation of 89%. Breath sounds are diminished and wheezing is absent. Based on this intervention, the nurse should anticipate interventions to treat which acid-base imbalance? a. Respiratory acidosis b. Respiratory alkalosis c. Metabolic acidosis d. Metabolic alkalosis

A. (Rationale: The nurse should anticipate treatment of respiratory acidosis. The assessment data indicates status asthmaticus with respiratory distress despite treatment. Even though the respiratory rate is elevated, the nurse should recognize that bronchoconstriction has caused CO2 retention. If the child was able to blow off the excess CO2 with hyperventilation, this would cause respiratory alkalosis. Option C and option D are incorrect because the child has an obvious respiratory problem and these should be eliminated from consideration immediately.)

The client has just experienced a 90-second grand mal seizure and has the arterial blood gas values shown below. How should the nurse be prepared to intervene? pH 6.88, HCO3- 22 mEq/L, PCO2 60 mm Hg, PO2 50 mm Hg A. Applying oxygen by mask or nasal cannula B. Applying a paper bag over the client's nose and mouth C. Administering 50 ml of sodium bicarbonate intravenously D. Administering 50ml of 20% glucose and 20 unites of regular insulin

A. (Rationale: The nurse should interpret this ABG as respiratory acidosis related to hypoventilation. However, the priority should be on the PO2 level, which in an ABG should be between 80-100 mm Hg so the nurse should prioritize placing the patient on oxygen. Option B is an intervention for respiratory alkalosis. Option C would be an intervention related to metabolic acidosis. Option D would be appropriate for treatment of hyperkalemia.)

the nurse has just received change of shift report. which patient should be seen first? A. a patient with neutropenia with a temp of 100.1 B. a patient with lymphoma and a blood pressure of 94/60 C. a patient with thrombocytopenia that has bleeding gums D. a patient with leukemia with white pharyngeal lesions

A. (rational: A patient with neutropenia has a limited ability to fight infection and activate an immune response. It is uncommon for this patient to generate a febrile temperature and anything greater than 100.4 is considered an emergency. This patient is also less likely to generate warmth, redness, and discharge at infectious sites. A blood pressure of 94/60mmHg is not a circulatory concern, this patient is maintain perfusion. A patient with thrombocytopenia is at risk for bleeding at multiple sites, however bleeding gums is not a high risk for perfusion. Internal bleeding would cause a higher priority. The patient with leukemia has thrush and while interventions are needed, they are not emergent.)

a client with sickle cell anemia is admitted to the hospital complaining of severe pain. which action will the nurse take first? A. initiate 0.9% sodium chloride at 200mL/hr B. administer one unit of PRBCs C. administer hydroxyurea (Droxia) D. prepare for a bone marrow transplantation

A. (rational: All of these are treatments for sickle cell anemia. However, the client in severe pain is likely to be in sickle cell crisis. To prevent further sickling of the red blood cells, adequate hydration of at least 200 mL/hr is needed during a crisis.)

The nurse has reviewed the patient's current blood gas analysis. Which value should the nurse communicate immediately to the healthcare provider? a. arterial oxygen tension (PaO2) of 60 mm Hg. b. arterial oxygen saturation (SaO2) of 91%. c. arterial carbon dioxide (PaCO2) of 47 mm Hg. d. arterial bicarbonate level (HCO3) of 27 mEq/L.

A. (rational: All the values are abnormal, but the low PaO2 indicates that the patient is at the point on the oxyhemoglobin dissociation curve where a small change in the PaO2 will cause a large drop in the O2 saturation and a decrease in tissue oxygenation. The nurse should intervene immediately to improve the patient's oxygenation.)

A patient with bacterial pneumonia has coarse crackles and thick sputum. Which action should the nurse plan to promote airway clearance? a. Help the patient to splint the chest when coughing. b. Encourage the patient to wear the nasal O2 cannula. c. Restrict oral fluids during the day. d. Encourage pursed-lip breathing technique.

A. (rational: Coughing is less painful and more likely to be effective when the patient splints the chest during coughing. Fluids should be encouraged to help liquefy secretions. Nasal O2 will improve gas exchange but will not improve airway clearance. Pursed-lip breathing can improve gas exchange in patients with chronic obstructive pulmonary disease but will not improve airway clearance.)

A patient whose lung cancer has metastasized to the bone reports lethargy, nausea, and vomiting. the healthcare provider suspects the patient is experiencing hypercalcemia. which of the following assessments associated with hypercalcemia would confirm this suspicion? A. decreased deep tended reflexes B. presence of Chvostek's sign C. diarrhea D. hypotension

A. (rational: Hypercalcemia leads to decreased deep tendon reflexes. Chvostek's sign is associated with low calcium levels. Diarrhea is a common cause of electrolyte imbalance. High calcium levels often correspond with increased blood pressure.)

After receiving change of shift report, which of the following patients will the nurse attend to first? A. A 45-year-old male undergoing treatment for non-Hodgkins lymphoma with a potassium level of 7.5mEq/L B. A 16-year-old female with sickle cell reporting a pain of 6 on a numerical pain scale C. A 55-year-old female with ovarian cancer waiting to be discharged D. A 67-year-old male with chronic obstructive pulmonary disease (COPD) and an oxygen saturation of 90% on room air

A. (rational: Normal serum potassium is 3.5-5mEq/L, the patient with a potassium level of 7.5mEq/L is at risk for cardiac complications. The patient with pain will need intervention, however this priority falls below respiratory and circulatory problems. A patient awaiting discharge will need the RN to complete this task when available. This answer does not indicate a need for emergent action. A COPD patient has a therapeutic oxygen saturation of 88-92%, this patient is normal and the nurse should continue to monitor.)

the older adult client is receiving the third unit of packed red blood cells that have been administered in the last 8 hours. one hour into the third transfusion, a nurse observes the client to have distended neck veins in the sitting position. what is the nurse's best first action? A. slow the infusion rate B. discontinue the infusion C. document the observation as the only action D. check the type of infusing blood with the client's blood type

A. (rational: Older adult clients are at risk for developing fluid overload during transfusion therapy, especially when receiving multiple units of packed red blood cells, which have a high osmotic pressure. The infusion rate should be slowed as low as possible, even before the client's circulatory status is assessed, to prevent worsening of the problem.)

The nurse is reviewing the patients home medication list. The patient had well controlled blood sugars with glyburide daily, but has recently had fast blood glucose levels between 180-200 mg/dl. Which medication may be contributing to the hyperglycemia? a. Prednisone b. Atenolol c. Phenelzine d. Allopurinol

A. (rational: Prednisone may decrease the effects of oral hypoglycemics as well as insulin by making the liver resistant to insulin. Patients on both prednisone and insulin or antidiabetic agents should closely monitor their blood sugars and watch for signs and symptoms of hyperglycemia.)

the client is scheduled for surgery and has expressed concern that she might received blood products, an act prohibited by her religion. the nurse will appropriately respond with which statement? A. there are other ways to replace blood loss besides blood products B. your chance of needing a blood transfusion is small C. you need to do what is necessary to save your life D. you could have family members donate blood

A. (rational: The client's rights and wishes should be respected while providing accurate information for reassurance. Directed donations from family members neither ensure safe blood products nor may be sanctioned by the client's religion.)

The nurse is admitting a patient who has a diagnosis of an acute asthma attack. Which information obtained by the nurse indicates that the patient may need teaching regarding medication use? a. The patient has been using the albuterol (Proventil) inhaler more frequently over the last 4 days. b. The patient became very short of breath an hour before coming to the hospital. c. The patient has been taking acetaminophen (Tylenol) 650 mg every 6 hours for chest-wall pain. d. The patient says there have been no acute asthma attacks during the last year.

A. (rational: The increased need for a rapid-acting bronchodilator should alert the patient that an acute attack may be imminent and that a change in therapy may be needed. The patient should be taught to contact a health care provider if this occurs. The other data do not indicate any need for additional teaching.)

the nurse is reviewing the following labs: RBC: 3.5 hemoglobin: 10.6 hematocrit: 32.1 platelets: 157 WBC: 9.5 which action by the nurse will be most therapeutic? A. administer erythropoietin (Epogen) B. starting an IV of 0.9% sodium chloride C. administering one unit of platelets D. giving subcutaneous injection of vitamin K

A. (rational: The natural stimulus for continued production of erythrocytes is the presence of erythropoietin. An injection of erythropoietin will help the bone marrow increase the production of erythrocytes. Saline, platelets, and vitamin K have no influence on making the bone marrow increase its production of erythrocytes.)

The nurse on the intermediate care unit received change-of-shift report on four patients with hypertension. Which patient should the nurse assess first? a. 48-yr-old with a blood pressure of 160/92 mm Hg who reports chest pain b. 52-yr-old with a blood pressure of 198/90 mm Hg who has intermittent claudication c. 50-yr-old with a blood pressure of 190/104 mm Hg who has a creatinine of 1.7 mg/dL d. 43-yr-old with a blood pressure of 172/98 mm Hg whose urine shows microalbuminuria

A. (rational: The patient with chest pain may be experiencing an acute myocardial infarction and rapid assessment and intervention are needed. Intermittent claudication, elevated creatinine and microalbuminuria show target organ damage but do not indicate acute process.)

the nurse is preparing to teach the family of a 4-year-old with hemophilia how to promote coagulation. which of the following instructions is most important to include? A. immobilize the joint B. apply a warm moist compress C. ensure patient receives annual influenza vaccine D. increase fluid intake

A. (rational: The patient with hemophilia is at risk of prolonged bleeding. Interventions for coagulation include immobilizing an affected joint to promote coagulation and hemostasis. A warm compress will promote circulation to the area, potentially increasing the bleeding. Maintaining the annual influenza vaccine will not impact bleeding. Recommending a decrease in fluid intake puts the patient at risk for FVD and would increase perfusion risks during times of bleeding.)

a patient is about to undergo allogenic bone marrow transplantation. which statement by the patient indicated a need for additional education? A. which bone will the surgeon insert the marrow in B. until the marrow transplant takes I should have few visitors C. the transplant does not start working immediately D. I will need chemotherapy prior to my transplant

A. (rational: The transplanted marrow is delivered intravenously. It is not placed into any bone. The client is at risk for infection until the bone marrow begins producing white blood cells. Therefore, visitors should be limited to prevent infection to the client. Engraftment, or the successful take of the transplanted cells, takes anywhere from 8 to 28 days, depending on the type of cell transplantation. In order for the donated marrow or stem cells to work, the client needs large doses of chemotherapy prior to transplantation.)

the nurse is assessing a patient with a new diagnosis of anemia. which symptom needs to be communicated to the provider immediately? A. dyspnea at rest B. Blood pressure of 150/70 C. Heart rate of 45 beats/min D. Pallor

A. (rational: anemia is a reduction in the number of RBCs amount of hemoglobin or hematocrit level. tissue oxygenation depends on RBCs. typical symptoms of anemia patients include dyspnea, increased somnolence, tachycardia, and pallor. a patient who is anemic tends to have a low blood pressure, increased heart rate, and skin that is pale)

the nurse is planning care for a patient admitted to the hospital for treatment of active TB. the nurse understands tht the patient is admitted to the hospital for which reason? A. evaluate the patients condition B. assess their compliance with the treatment plan C. to prevent the spread of the disease to others D. to determine the need for antibiotic therapy

A. (rational: the reason for hospitalization is based on physiologic needs. With TB, a change in oxygenation status would be the largest reason to be admitted to the hospital.)

The nurse assesses the client who presents to the emergency department with a panic attack. Which findings should prompt the nurse to ask the healthcare provider about obtaining ABGs? SELECT ALL THAT APPLY A. Respirations 40 breaths/minute B. Tingling in the fingers C. Muscle twitching D. Salivation E. Increased urination

A. B. C. (Rationale: Respiratory alkalosis often occurs with panic attacks as hyperventilation blows off CO2 leading to acid loss. Option A, the respiratory rate of 40, should be reported. Option B, tingling in the fingers, occurs in respiratory alkalosis due to the increase in neuromuscular excitability associated with hyperventilation. For this same reason, Option C, muscle twitching, should also be reported as it indicates increased neuromuscular excitability. Option D, salivation, is not associated with respiratory alkalosis from a panic attack. Option E, increased urination, could occur as part of the stress response but it not directly related to respiratory alkalosis.)

A patient is admitted with a venous stasis ulcer. Which assessment findings are consistent with this diagnosis? SELECT ALL THAT APPLY. a. Superficial with irregular margins b. Brown discoloration c. Cool to touch d. Severe pain e. Palpable pedal pulses

A. B. E. (Rationale- Cool temperature of skin as well as pain indicate an arterial issue not venous. This is directly related to lack of blood flow to the peripheries, which causes tissue ischemia. Arterial ulcers also accompany deep and rounded margins with normal to atrophic appearance in color.)

The nurse is teaching a patient with heart failure about dietary restrictions. Which food choices made by the patient indicates a need for further teaching? SELECT ALL THAT APPLY A. Can of low fat, chicken noodle soup B. Ham sandwich on a whole-grain bun C. Grilled salmon with potatoes D. Fresh fruit cup E. Stir fry vegetables with soy sauce

A. B. E. (Rationale: The selected food items are processed foods which contain a lot of sodium. Fresh fruits, fish and potatoes are not processed and would not include a lot of sodium.)

a charge nurse is making room assignments which two patients should be assigned to the same room? Select two A. a patient three days post laryngectomy that is receiving tube feeding B. a patient with pneumonia and is influenza A positive C. a patient post lung resection surgery with a chest tube D. a new admission with a diagnosis of rule out TB

A. C. (rational: These are the two patients that don't possess a risk of causing an infection to another patient or are not at greater risk of getting an infection.)

The healthcare provider wants to attempt the use of an NSAID for a patient with chronic pain. Which medications can the nurse choose from the MAR? Select all that apply. a. Aspirin b. Acetaminophen c. Ketorolac d. Naproxen

A. C. D. (Rationale: Tylenol is the only one that is not classified as an NSAID)

A patient is admitted with a long history of alcohol abuse. Which lab values would the nurse anticipate? Select all that apply A. Calcium 7. B. Albumin 4.0 C. Magnesium 3.2 D. BUN 33 E. Potassium 2.3

A. C. E. (Rational: Alcoholics often present with multiple electrolyte deficiencies related to poor dietary intake. Option A is correct because chronic alcohol users have many risk factors for hypocalcemia. First dietary intake of calcium and vitamin D is low. In addition, damage to the liver due to alcohol use causes deficiencies in the enzyme needed to convert vitamin D to its active form. Option C is correct because magnesium is chronically depleted in alcoholics due to poor nutrition. Alcohol also causes increased excretion of magnesium via the urine. Approximately 90% of patients experiencing alcohol withdrawal will have hypomagnesemia. Option E is correct because chronic alcoholics often experience hypokalemia related to a variety of factors. These can include withdrawl, poor nutrition, GI losses due to diarrhea or upset stomach, chronic pancreatitis and concurrent low magnesium levels. Option B is incorrect because albumin levels in chronic alcoholics are typically low due to poor dietary intake; specifically, poor protein intake. Option D is incorrect because BUN levels in chronic alcoholics are typically low. Elevated BUN is seen in severe fluid volume deficit or kidney injury)

The nurse is initiating a patient-controlled analgesia (PCA) pump of morphine for a patient with acute surgical pain who has never received opioids. Which information should the nurse include in the patient's care plan? (SELECT ALL THAT APPLY). a. Provide education about how pain management improves post-operative activity levels b. Assess for signs of withdrawal such as diarrhea, abdominal pain and sweating c. Monitor the patient for confusion, sedation and itching d. Review consequences of addiction with the client e. Teach the client about the plan for discharge including the transition to oral medication

A. C. E. (Rationale: A client who is opioid-naïve is at higher risk of adverse effects when administering IV opioids. The nurse should provide education about how to use the pump effectively. Option A, provide education about how pain management improves post-operative activity levels, is correct as this education will help the client to understand the connection between pain management and mobility associated complications such as skin breakdown and respiratory complications. Option C, Monitor the patient for confusion, sedation and itching, is correct because these are the common adverse effects associated with morphine and the client who has never had them should be monitored closely. Option E, Teach the client about the plan for discharge including the transition to oral medication, is correct as the client should be aware that the PCA is only for the acute pain phase and that once their GI tract is functioning, they will be transitioned to oral medication. This is the only option for them to go home with as well. Option B, Assess for signs of withdrawal such as diarrhea, abdominal pain and sweating, is incorrect as the client should not experience any signs of withdrawal at this point in their care. Withdrawal is typical with continuous opioid use and usually begins 2-10 days after abruptly stopping an opioid medication. Option D, Review consequences of addiction with the client, is incorrect because this client is not at risk for developing addictive behaviors because they are opioid naïve. The nurse should focus their education on safe use of the PCA pump. Discussing addiction with the patient at this point may be detrimental as they may be fearful to use the medication to control their pain.)

The nurse is prepating to administer lisinopril, digoxin, and carvedilol for a patient with heart failure. The nurse reviews the patient's chart for the following data: Blood pressure- 110/70 Heart rate- 48 Respiratory rate- 16 O2 sat- 98% Temp- 99.2 oral Sodium0 145 Potassium- 4.8 Magnesium- 1.8 calcium- 8.4 BUN- 18 creatinine- 1.1 Which action should the nurse take? SATA A. administer lisinopril B. hold the digoxin and administer the lisinopril and carvedilol C. hold the digoxin and the carvedilol D> contact the health care provider regarding administering the digoxin E. contact the health care provider regarding administration of digoxin and carvedilol

A. C. E. (Rationale: Lisinopril is an ace-inhibitor and will lower the blood pressure. Digoxin is an inotropic medication that will decrease the heart rate and increase the force of the myocardial contraction. Carvedilol is a noncardioselective beta blocker and decreases the heart rate and blood pressure. In this case, the patient has a normal BP, low HR and normal potassium. Because the Digoxin and Carvedilol both decrease the HR, they should be held, and the health care provider should be notified. Lisinopril will not affect the HR, so it can be administered safely.)

the nurse is caring for a patient with pancytopenia. which assessment data is consistent with this diagnosis? SATA A. scattered bruises B. deep vein thrombosis C. dyspnea on exertion D. urine output 275mL in 8hrs E. thick yellow sputum

A. C. E. (rational: Patients with pancytopenia are low in RBC, WBC, and platelets. Bruises may be observed related to the low platelets. Low RBC will present with poor perfusion and may be observed as dyspnea, pallor, low activity intolerance, etc. The patient with low WBCs will have a weakened immune system, making them susceptible to infection.)

Glimepiride (Amaryl) is prescribed to a patient with type two diabetes. The nurse instructs the patient that which food items should be avoided while taking this medication? SELECT ALL THAT APPLY. a. Alcohol b. Red meats c. Whole-grain foods d. Low-calorie desserts e. Poultry

A. D. (rational: When alcohol is consumed with glimepiride the patient may complain of flushing, palpitations and nausea. Alcohol may accelerate hypoglycemic effects. Therefore, the patient should be instructed to avoid alcohol. Low-calorie desserts should also be avoided; even though the calorie count is low, carbohydrate count is likely high and can affect the blood glucose level.)

a patient is admitted with a WBC of 0.6 and an absolute neutrophil count of 0.3. the nurse will complete which of the following actions to promote patient safety? select all that apply A. check oral temperature every four hours B. avoid subcutaneous injections C. assess wounds for redness and drainage D. ensure fruits and vegetables are cooked E. Educate on proper hand hygiene F. administer filgrastim (Neupogen) as prescribed

A. D. E. F. (rational: Nursing care for patients with anemia should alternate periods of rest and activity to maintain patient mobility without causing undue fatigue. High vitamin K diets might be used for a patient with a bleeding disorder. There is no indication that the patient is neutropenic, so isolation is not needed. Increased intake of fluid and fiber will not improve the anemia.)

A pregnant woman is started on IV magnesium sulfate for treatment of pre-eclampsia. The nurse notes the following rhythm in lead 3: Which lab value correlates with this EKG finding? select all that apply A. K+ of 6.3 B. K+ of 2.9 C. Ca++of 8.8 D. Mg++ of 2.1 E. Mg++ of 3.5

A. E. (Rational: IV magnesium is the treatment of pre-eclampsia and patients should be closely monitored for electrolyte imbalances. In this scenario, the patient is getting IV magnesium so hypermagnesia should be expected. In addition, hypermagnesia results in hyperkalemia as well. The EKG shows tall, peaked T waves which also correlate with hypermagnesmia and hyperkalemia. Hyperkalemia affects cardiac conduction by inducing tall T waves, widening QRS complexes, absent P waves, prolonged PR intervals, bradycardia, and heart block. Option B is hypokalemia which would not occur in this scenario and would cause flat T waves and the development of a U wave. Option C is normal calcium level and would not be expected in this scenario or cause these EKG changes. Option D represents a normal magnesium level)

A nurse is caring for a female client in the emergency department who reports shortness of breath and pain with a deep breath. The patient states she started taking birth control 3 weeks ago. The nurse collects the following assessment data: HR: 110bpm RR: 40breaths/min BP: 140/80 pH: 7.50 PaCO2: 29 PaO2: 60 HCO3: 20 SaO2: 86% Which intervention should the nurse do first? a. Prepare for mechanical ventilation b. Administer oxygen via face mask c. Prepare to administer a sedative d. Draw blood for a d-dimer and prepare patient for CT scan

B. (Rational: The nurse interprets and filters the abnormal data in this scenario first. The patient is tachycardic, tachypneic, hypoxic and is experiencing respiratory alkalosis. Next, once the data has been organized, the nurse should prioritize an action. Using the ABC framework, the nurse should realize that the most urgent problem is related to breathing and hypoxia. The first action should be to apply oxygen. Option A is incorrect because this is not the best FIRST action. If her condition declines or conservative treatment does not help, this may be necessary. Option C is incorrect because the patient's hypoxia is the priority using the ABC framework. In many instances, respiratory alkalosis is caused by anxiety and blowing off excess CO2 with hyperventilation and this may be the case for this patient as well, but her oxygen level should be correct first. Option D is incorrect because this is not the first action. The nurse should use the ABC framework and while a PE may indeed be the cause of this condition, treating the patient's low O2 level is a priority before investigating the cause.)

The nurse is preparing to administer 5%NaCl intravenously. Which lab value should the nurse evaluate prior to administration? A. Potassium B. Sodium C. Calcium D. Chloride

B. (Rational: 5%NaCl is a hypertonic solution and will draw fluid into the intravascular compartment. This is typically for the treatment of hyponatremia. In hyponatremia there are fewer solutes (sodium) than water. This is done by administering hypertonic solution. Administering hypertonic solutions requires precise trending of labs; specifically sodium level. Correcting the sodium level too rapidly can cause neurological deficits including cerebral edema. Ideally a sodium level shouldn't be corrected quicker than 1-2 mmol/L per hour. For example, if the baseline sodium is 122 mmol/L in three hours the sodium shouldn't exceed 128 mmol/L.)

A patient is admitted with fluid volume overload due to CKD. Which assessment finding indicates that the treatment was effective? A. serum sodium level of 133mEq/L B. Weight change from 78kg to 75kg C. urine specific gravity change from 1.026 to 1.025 D. Net I&O for the hospitalization of +200ml

B. (Rational: An increased urine output coupled with weight loss is good indication that overhydration is resolving. A serum sodium level of 133 mEq/L is low and may reflect dilutional hyponatremia. A change in urine specific gravity by one unit may indicate the beginning of a trend but itself is not significant. A narrowing of pulse pressure may indicate the beginning of fluid loss or may be a result of cardiac complications. Again, by itself, the pulse pr3essure change does not support resolution of the overhydration)

A patient admitted to a medical unit is NPO and is receiving D5W intravenously. They are now confused to time and place. Which electrolyte imbalance could support this change in condition? A. Hyperkalemia B. Hyponatremia C. Hypocalcemia D. Hypernatremia

B. (Rational: Hyponatremia is associated with confusion, especially in the elderly population. This is because there is a deficit of sodium in the extracellular space causing a shift of water into the intracellular space. Remember where there is a higher concentration of salt, water will follow. In extreme cases neurological deficits including cerebral edema will occur as excess water causes the cerebrum to swell. Additionally, D5W contains not electrolytes. With the patient being NPO hyponatremia could manifest)

The physician orders 0.45%NaCl at 125ml/hr for a patient undergoing a prostatectomy. The nurse hangs the 1l bag at 1000. What time should the nurse anticipate needs a new bag of IV fluid? A. 1600 B. 1800 C. 2030 D. 2100

B. (Rational: a one liter of fluid is 1000ml. The nurse should then determine how long it will take to infuse 1000ml. This is done by dividing 1000 by the prescribed rate of 125ml/hr. This comes to 8hrs. Then the nurse will add 8hrs to 1000 to arrive at the correct response of 1800)

A patient complains of claudication after walking a distance of one block. The nurse notes that the patient has developed a painful ulcer on the toes of the right foot. Which condition is most likely responsible for this patient's symptoms? a. Diabetic foot ulceration b. Peripheral arterial disease c. Peripheral venous disease d. Necrosis or gangrene of the toes

B. (Rationale- Arterial disease is characterized by claudication after walking short distances. Ulcerations caused by peripheral arterial disease are painful and initially located at the most distal points on the extremity. Diabetic ulcers and venous ulcers are seldom painful and usually tend to occur where pressure is applied.)

Which breakfast food recommendations would be most appropriate for a patient who has been placed on a low-cholesterol diet? a. Eggs, skim milk, whole wheat toast, decaffeinated coffee b. Skim milk, cereal, banana, decaffeinated coffee c. Toast margarine, one slice of bacon, coffee d. Blueberry muffin, orange juice, coffee

B. (Rationale- Cholesterol is found in animal-based products, such as milk, eggs, and cheeses. These ingredients are also found in baked goods, such as muffins)

A patient with a diagnosed with a AAA suddenly develops sharp lower back pain radiating to the side. What is the nurse's interpretation of this information? A. the aneurysm has become obstructed B. the aneurysm may be undergoing expansion and/or possible rupture C. the client is experiencing inflammation of the aneurysm D. the client is experiencing normal sensation associated with this condition

B. (Rationale- Many patients experience no symptoms of their AAA. An aneurysm rupture is characterized by sharp back pain, pain that radiates to the side and possibly the lower pelvis, an elevated WBC, and flank bruising.)

After undergoing an arteriogram via the femoral approach, the patient has a large puddle of blood under his buttocks. What step should the nurse take first? a. Ask the patient to logroll to the side b. Apply gloves and assess the groin site c. Call the provider d. Obtain vital signs

B. (Rationale- The femoral artery is the source of access for the arteriogram. If bleeding is occurring at the site, immediate compression must take place. Vital signs are important, but will be taken after hemostasis has been established. Rolling the patient at this time is not appropriate because they could still be bleeding from the site. The provider notification may be appropriate, but not until the side of the bleed has been assessed and vital signs obtained.)

A nurse is educating a client who is postoperative following coronary artery bypass graft (CABG) surgery and is receiving acetaminophen/hydrocodone for discomfort. Which additional benefit is most important for the nurse explain to the patient? a. "This medication will also help to decrease your level of anxiety." b. "This medication will help you to take deep breaths and cough." c. "This medication will make it easier for you to sleep in the hospital." d. "This medication will help to reduce your blood pressure."

B. (Rationale: Clients recovering from a CABG surgery will often have their pain managed with opioid medication. The nurse should prioritize coughing and deep breathing as an important postoperative intervention. Opioid medications help to minimize the discomfort experienced with coughing and deep breathing following a chest surgery as well as increase compliance. These exercises help to prevent the development of postoperative pneumonia. Option A is incorrect because even though opioids can reduce anxiety and apprehension, the nurse should explain the breathing benefits of opioids. Option C is incorrect because opioids can induce rest and sleep in a postoperative patient however, the most important explanation is related to breathing. Option D is incorrect because even though opioids can reduce blood pressure, this is not typically a desired effect and the nurse should closely monitor the cardiac status of the post-operative patient.)

The nurse is caring for a client with respiratory acidosis. Which ABG results indicate to the nurse that the client's kidneys have fully compensated for the imbalance? A. pH 7.32, PACO2 48 mm Hg, HCO3 35 mmol/L B. pH 7.35, PACO2 50 mm Hg, HCO3 30 mmol/L C. pH 7.33, PACO2 55 mm Hg, HCO3 24 mmol/L D. pH 7.44, PACO2 45 mm Hg, HCO3 24 mmol/L

B. (Rationale: In respiratory acidosis, the client is experiencing hypoventilation, leading to a decrease in pH and an increase in PaCO2. Over time, the kidneys will begin to balance but this can take 72 hours or more. On an ABG, this will cause the HCO3 level to increase as the kidneys slowly retain HCO3 to counteract the acid level. Full compensation occurs when the pH returns to normal. Remember, if HCO3 is outside of the normal range but the pH is still abnormal, this is only partial compensation. The correct response is B, pH returned to normal, PaCO2 is elevated and so is the bicarb level indicating kidney compensation. Option A is respiratory acidosis with partial compensation (the pH is abnormal) Option C is uncompensated respiratory acidosis Option D is a normal ABG)

A patient with extensive second-degree burns on the legs and trunk is using patient-controlled analgesia (PCA) with IV morphine to be delivered at 1 mg every 10 minutes to control the pain. Several times during the night, the patient awakens in severe pain, and it takes more than an hour to regain pain relief. Which action should the nurse take? a. Request that the health care provider order a bolus dose of morphine to be given when the patient awakens with pain. b. Consult with the patient's health care provider about adding a continuous morphine infusion to the PCA regimen at night. c. Teach the patient to push the button every 10 minutes for an hour before going to sleep even if the pain is minimal. d. Administer a dose of morphine every 1 to 2 hours from the PCA machine while the patient is sleeping.

B. (Rationale: It is the responsibility of the nurse to effectively manage the dosing and administration of the PCA. In this scenario, the patient's pain is not controlled with the current settings. The nurse has gathered information in this case and notes that this is only happening at night when the patient cannot push the button. For this reason, the nurse should request a continuous or basal dose of medication. This will run in the background, whether the patient is awake or sleeping, and allow for better pain control without the peaks and valleys. Option A is incorrect because once the pain is out of control, it typically requires more frequent dosing or larger amounts of medication to regain control. The nurse should plan an intervention that will prevent the pain from becoming severe. Option C is incorrect because this incorrect instruction for the patient and could lead to adverse effects. Patients should be taught to push the button when they have pain and not to wait until it becomes intense however if they do not have pain, they do not need to push the button. Option D is incorrect as this action is unethical because the patient is sleeping. PCA machines should only be used by the patient and the nurse should not dose medication, unless ordered and requested by the patient, from the machine.)

The nurse is evaluating a patient with left-sided heart failure after the administration of bumetanide (Bumex). Which finding indicates that the treatment was effective? A. Urine output of 250ml over 8-hour shift B. Lung sounds demonstrate improved aeration C. Edema consistent at 2+ to lower extremities D. Level of consciousness declined

B. (Rationale: Left-sided heart failure affects the lungs so they would be the most significant thing to assess after the administration of a diuretic. LOC can be affected by a lot of factors. Edema is more pertinent to the right side of the heart. And the question is referring to left-sided heart failure as the primary concern instead of low urine output.)

The nurse is caring for a patient with several broken ribs. The nurse should be alert for which complication? a. Respiratory alkalosis from anxiety b. Respiratory acidosis from inadequate ventilation c. Metabolic acidosis from calcium loss from broken bones d. Metabolic alkalosis from ingestion of base-containing analgesics

B. (Rationale: Pain from broken ribs often causes the client to breathe more shallowly to avoid moving his or her ribs and increasing pain. If respiration is shallow enough, ventilation is inadequate, leading to poor gas exchange and respiratory acidosis.)

The nurse is caring for a client who is taking naproxen for rheumatoid arthritis. Which statement by the patient should the nurse investigate further? a. "I signed up for a swimming class.: b. "I've been taking a daily antacid to help with indigestion." c. "I've lost 2 pounds since my appointment 2 weeks ago." d. "The naproxen is easier to take when I crush it in applesauce."

B. (Rationale: Rheumatoid arthritis is a painful autoimmune condition that is often manage with NSAIDs. NSAIDS can cause serious GI adverse reactions such as ulceration and bleeding. The patient should be taught that any GI upset, burning, vomiting or blood in the stool while taking and NSAID should be reported promptly to the provider. The nurse should investigate this statement further to determine a potential cause. Option A is incorrect because daily exercise, especially low-impact exercise like swimming, is a great intervention to relieve stiff muscles and maintain range of motion. Option C is incorrect because this rate of weight loss is acceptable and weight management can be helpful in the treatment of rheumatoid arthritis because it decreases stress on the joints. Option D is incorrect because naproxen can be swallowed whole or crushed.)

The nurse is caring for a patient with a recent myocardial infarction. Which statement by the nurse best describes the damage that has occurred? A. "Damage to this area causes a buildup of plaque on the heart valves." B. "Damage to this area can decrease the strength of each contraction of the heart muscle. C. "Damage to this area can lead to kidney failure" D. "Damage to this area causes heart muscle fibers to release damaging high-density lipoproteins"

B. (Rationale: The myocardium is the layer responsible for the contractile force of the heart.)

The nurse in the emergency department is caring for a patient who was choking on a piece of steak. The nurse notes dyspnea, chest discomfort and the patient reports a "lump in my throat" with a hoarse voice. The nurse receives the following blood gas report: pH: 7.12 PaCo2: 52 mEq/L HCO3: 24 mm Hg PaO2: 72 mm Hg Which intervention should the nurse complete first? a. Explain to the patient the need for flexible bronchoscopy b. Apply oxygen via simple mask c. Assess for vocal fremetis d. Prepare to intubate the patient

B. (Rationale: The scenario presented here is emergent - a patient who was choking and now has chest discomfort and a hoarse voice is most likely experiencing an obstructed airway. The nurse should use the ABC framework to guide decisions. In this type of question, "what should the nurse do first," this is the correct framework to do because all options will be viable to the situation but you must pick the first, most urgent, prioritized answer. After interpreting the ABG as respiratory acidosis with hypoxemia, the nurse has gathered enough assessment data to begin an intervention. Option B, apply oxygen via simple mask, is correct because this option uses the ABC framework and addresses the immediate oxygen needs evidenced by the low PaO2. Option A, explain to the patient the need for flexible bronchoscopy, is incorrect because this is a teaching intervention and would not take priority over breathing concerns. While a flexible bronchoscopy is the likely treatment, education on this topic would not be a priority. Option C, assess for vocal fremetis, is incorrect because this assessment is not needed at this point. The nurse has gathered enough information and assessing for vocal fremetis will not offer any additional insight into the decision making. Option D, prepare to intubate the patient, is incorrect as this is not the first action. If the patient were to deteriorate or move to respiratory distress, then intubation may be necessary. However, the patient is alert, talking and while they have discomfort, they are breathing independently and do not require intubation at this time.)

a patient with heart failure is being discharged home. which topics should the nurse include in the discharge teaching plan? A. diet managment, psychosocial concerns and medicine B. medications, activity, weight, diet, and symptoms C. medications, diet restriction, weight reduction, and financial concerns D. weight management, symptoms, social support, spirituality, and medications

B. (Rationale: When the nurse is providing education to the patient, the answer B has the highest priority items listed. They will need education on the different medications they will be taking, their level of activity, how to track their weight, dietary restrictions and symptoms to report.)

the nurse is caring for a patient with left sided heart failure. which assessment findings should the nurse report to the provider immediately? A. anorexia B. dyspnea C. peripheral edema D. weight gain

B. (Rationale: With left sided heart failure, the left ventricle is not able to match the output of the right ventricle. This results in blood backing up into the lungs and leads to pulmonary congestion. Anorexia and Peripheral edema are associated with right sided heart failure. Left and right sided heart failure may cause weight gain.)

the nurse is caring for a patient admitted with a pulmonary embolism and started on heparin gtt. which of the following lab results indicates to the nurse that heparin therapy is therapeutic? A. platelet count is 356,000 B. PTT is 56 seconds C. INR is 2.1 D. Hemoglobin is 9.6

B. (rational: A normal PTT is 25-35 seconds. When a patient is on anticoagulation, this number should be higher. Goal PTT for heparin therapy is 45-70 seconds. Heparin will not directly impact the PT/INR. Heparin therapy may decrease the platelet count as well as hemoglobin if the value is supratherapeutic.)

the nurse is performing the preoperative checklist on a patient. the patient indicated that they have been taking aspirin 325mg every day for pain and their last dose was yesterday. what would be a priority action for the nurse? A. document the time of last administration and take no further action B. contact the surgeon C. administer subcutaneous vitamin K D. administer acetaminophen for pain

B. (rational: Aspirin and other salicylates interfere with platelet aggregation, the first step in the blood-clotting cascade, and decrease the ability of the blood to form a platelet plug. These effects last for more than 1 week after just one dose of aspirin. The client may need the surgery rescheduled. Vitamin K, prescribed pain medication, and Tylenol cannot reduce the anticlotting effects of aspirin.)

the nurse is evaluating a patient's response to propranolol. which assessment finding indicated the patient is experiencing an unexpected outcome? A. insomnia B. audible expiratory wheezes C. a blood pressure of 124/72 after medication administration D. a heart rate of 78 after medication administration

B. (rational: Audible expiratory wheezes may indicate a serious adverse reaction; bronchospasm. Beta blockers may induce this reaction especially beta 2 blockers, which have a systemic affect on the body. The blood pressure and heart rate are expected findings after medication administration. Insomnia is a mild side effect and should be monitored)

The nurse has instructed a patient on their prescribed corticosteroid. Which statement indicates that the client understands teaching about the correct use of this medication? a. "This drug can reverse my symptoms during an asthma attack." b. "This drug is effective in decreasing the frequency of my asthma attacks." c. "This drug can be used most effectively as a rescue agent." d. "This drug can safely be used on a long-term basis for multiple applications daily."

B. (rational: Corticosteroids decrease inflammatory and immune responses in many ways, including preventing the synthesis of mediators. Both inhaled corticosteroids and those taken orally are preventive; they are not effective in reversing symptoms during an asthma attack and should not be used as rescue drugs. Systemic corticosteroids, because of severe side effects, are avoided for mild to moderate intermittent asthma and are used on a short-term basis for moderate asthma.)

A patient presents to the emergency department with confusion, acetone breathe, and kussmaul's respirations. Which arterial blood gas values indicate to the nurse that the client is experiencing diabetic ketoacidosis? a. pH: 7.38, HCO3 22 mEq/L, PaCO2 38 mmHg, PO2 98 mmHg b. pH: 7.28, HCO3 18 mEq/L, PaCO2 28 mmHg, PO2 98 mmHg c. pH: 7.48, HCO3 28 mEq/L, PaCO2 38 mmHg, PO2 98 mmHg d. pH: 7.28, HCO3 22 mEq/L, PaCO2 58 mmHg, PO2 88 mmHg

B. (rational: Diabetic ketoacidosis is a metabolic condition. Therefore, we are looking for a PH of < 7.35 , and a HCO3 of <22. This can be with or without compensation of the respiratory system. Remember the respiratory system can attempt to compensate in seconds to minutes. However, with kussmaul's respirations the effort of the respiratory system is poor and attempt to compensate may not be sufficient enough to normalize the pH level. Answer B is the only option with a low or acidotic HCO3 level and in this case there is partial respiration compensation.)

A patient is admitted with a hematocrit (Hct) of 32%. When taking the patient's history, which information is most significant? A. Use of acetaminophen (Tylenol) B. Consumption of alcohol C. Use of oxygen D. Recent infections

B. (rational: Diet can alter cell quality and affect blood clotting. Chronic alcoholism can cause nutritional deficiencies and liver impairment, both of which reduce blood clotting and cause bleeding, which would decrease the hematocrit.)

A patient with type 2 diabetes mellitus is admitted with confusion, dry mucous membranes and poor skin turgor. The serum sodium is 149 mEq/L; blood pressure 88/58; pulse 118 and serum glucose 465 mg/dl. Which priority action will the nurse complete first? a. Calcium gluconate b. Intravenous fluids c. Potassium IV drip d. Intravenous insulin

B. (rational: Intravenous fluids is most appropriate for this patient as the symptoms of dehydration (high serum sodium level, dry mucous membranes, and poor skin turgor, hypotension and tachycardia) are apparent. Calcium gluconate or potassium are not safe to administer as this is no data on these laboratory values. Intravenous insulin would be appropriate after administration of fluids.)

The nurse provides discharge instructions to a patient who was hospitalized for pneumonia. Which statement by the patient indicates a good understanding of the instructions? a. "I will schedule two appointments for the pneumonia and influenza vaccines." b. "I will continue to do deep breathing and coughing exercises at home." c. "I will cancel my follow-up chest x-ray appointment if I feel better next week." d. "I will call my health care provider if I still feel tired after a week."

B. (rational: Patients should continue to cough and deep breathe after discharge. Fatigue is expected for several weeks. The pneumococcal and influenza vaccines can be given at the same time in different arms. A follow-up chest x-ray needs to be done in 6 to 8 weeks to evaluate resolution of pneumonia)

The nurse administers the first dose of an antineoplastic to a client on the oncology unit. The nurse knows that which of the following activities is appropriate to delegate to the LPN? A. Provide teaching about the side effects of the medication B. Obtain the patient's blood pressure C. Administer the second dose of the oral medication D. Flush the patient's central line with heparin

B. (rational: The LPN may gather data regarding the blood pressure and report to the RN to assess. The LPN scope does not permit the action of assessing, evaluating, and education. The LPN will not be permitted to educate about a medication. Although LPNs can administer medications, answer C is pertaining to the antineoplastic medication in the stem. Only certified RNs may administer these. LPNs may not access and flush central lines.)

The nurse is caring for a patient having an asthma attack. Which Arterial Blood Gas results indicates that the patient is in an early phase of this condition? a. pH 7.0, PaCO2 50 mm Hg, and PaO2 74 mm Hg. b. pH 7.45, PaCO2 32 mm Hg, and PaO2 70 mm Hg. c. pH 7.36, PaCO2 40 mm Hg, and PaO2 80 mm Hg. d. pH 7.32, PaCO2 58 mm Hg, and PaO2 60 mm Hg.

B. (rational: The initial response to hypoxemia caused by airway narrowing in a patient having an acute asthma attack is an increase in respiratory rate, which causes a drop in PaCO2. The other PaCO2 levels are normal or elevated, which would indicate that the attack was progressing and that the patient is decompensating.)

the nurse assists the patient with anemia to ambulate in the hallway. which assessment finding indicates that the patient is not tolerating the activity? A. blood pressure is 106/90 B. heart rate is 120 beat/min C. oxygen saturation is 91% D. respiratory rate is 20 breaths/min

B. (rational: The red blood cells contain thousands of hemoglobin molecules. The most important feature of hemoglobin is its ability to combine loosely with oxygen. A low hemoglobin level can cause decreased oxygenation to the tissues, thus causing a compensatory increase in heart rate. The other distractors are close to normal range and are not indicative of not tolerating this activity.)

The nurse is caring for a patient admitted 1 hour ago with hyperglycemic, hyperosmolar, nonketotic syndrome (HHNS). Which order will the nurse question with the provider? a. Insulin Regular 0.1u/kg/hr IV continuous b. 0.9%NaCl with 20mEq/L Potassium Chloride at 750mL/hr c. Strict hourly I&O d. Telemetry

B. (rational: This is a hyperglycemic nonketotic syndrome and with HHNS potassium is likely used as additive with fluid replacement. An isotonic fluid like 0.9% normal saline is appropriate because as the glucose level increases, patients become extremely dehydrated (hence the name hyperosmolar) subsequently raising the blood glucose levels even higher as this is stress on the body. Isotonic solutions will disperse equally in the intra and extracellular spaces. Promptly correcting the dehydration will add in preventing higher blood glucose levels. A general protocol for HHNS is to administer 15-20 ml/kg/hr of fluid in the first hour. Potassium may need to be replaced but this will depend on the serum K level and it can be administered per the physician discretion either via a primary or additive.)

the nurse is planning care for a patient with CF. which test is mot diagnostic for this condition? A. serum sodium level B. skin sweat test C. fasting blood sugar level D. sputum for culture and gram stain

B. (rational: a skin sweat test of >60 is indicative of CF. The serum sodium level may be decreased with CF, but it is not as specific as the skin sweat test. Fasting blood sugars can be elevated, but there are numerous other causes of that as well. Sputum cultures would be used to diagnose pneumonia, a complication of CF.)

The nurse is caring for a patient post pneumonectomy for lung cancer that has decreased oxygen saturations. What action by the nurse at this time would be most appropriate? A. Position the patient in the lateral position with the nonsurgical side facing the bed B. Position the patient in the lateral position with the surgical side facing the bed C. Have the patient use a peak flow meter D. Implement chest physiotherapy with postural drainage

B. (rational: placing the remaining lung in the superior position will allow for better oxygenation because it will allow the remaining lung the ability to more fully expand. Peak flow is used for asthma attacks and chest physiotherapy and postural drainage are interventions used to keep an airway open.)

the nurse is assessing patients for their risk of developing laryngeal cancer. which of these patients should the nurse consider at the greatest risk? A. chews tobacco daily B. smokes one pack of cigarettes daily C. lives in a urban area D. consumes an alcoholic beverage the majority of days in the week

B. (rational: smokeless tobacco, cigarette smoking and alcohol are all risk factors of developing laryngeal cancer, but cigarette smoking is the biggest risk factor.)

a patient is prescribed INH, PZA, rifampin, and ethambutol for the treatment of TB. the nurse understands that they shouold be monitoring which lab value to assess for an averse reaction to the treatment? A. troponin B. AST C. ABG D. creatinine

B. (rational: the four drug therapy is most likely to lead to hepatotoxicity which would lead to an increase in AST and ALT labs.)

A patient is to receive 10 units of regular insulin for a blood glucose of 220 mg/dL. When would the nurse expect to see the onset of action for this medication? a. Less than 10 minutes b. In 30 to 60 minutes c. In one to three hours d. In six to twelve hours

B. (rational: the onset for regular insulin is 30-60 minutes, peak 2-12 hours, and duration is 24 hours.)

A patient has developed a deep vein thrombosis (DVT) in the left lower extremity and is started on IV heparin therapy. Which nursing interventions are appropriate for this patient? SELECT ALL THAT APPLY. a. Massage the leg b. Measure leg circumference c. Elevate extremity above the level of the heart d. Apply sequential devices (SCD's) to the affected extremity e. Monitor aPTT level

B. C. E. (Rationale- Massaging the leg and applying SCDS would be inappropriate interventions as this could aid in migrating the clot or form micro clots into the arterial system causing a PE or stroke. Measuring the leg circumference is appropriate for baseline assessment and future comparison. Elevating the extremity above the level of the heart is going to promote venous return. Monitoring an aPTT level is appropriate as this patient is on IV heparin. The aPTT should be 1.5 to 2x the normal value.)

the nurse is educating a client with cobalamin deficiency about appropriate dietary sources of this vitamin which items selected by the client indicates an appropriate understanding of this diet? SATA A. 1 glass of red wine B. fried liver C. grilled flat iron steak D. 1 medium apple E. 8oz whole milk

B. C. E. (rational: High dietary sources of cobalamin include meat (turkey, beef), poultry (chicken, eggs), seafood (clams, sardines), and dairy. Fruits, vegetables, and alcohol do not contain a high amount of cobalamin.) `

The nurse teaches the patient with diabetes mellitus about differentiating between hypoglycemia and ketoacidosis. The patient demonstrates understanding of teaching by stating that a form of glucose should be taken if which symptoms develop? SELECT ALL THAT APPLY. a. Polyuria b. Shakiness c. Palpitations d. Blurred vision e. Lightheadedness f. Fruity breath odor

B. C. E. (rational: Shakiness, palpitations and lightheadedness are signs and symptoms of hypoglycemia and indicate the need for food or glucose. Other answers are signs and symptoms of hyperglycemia.)

The nurse is assessing a patient following chemotherapy induction two days ago. The patient is having persistent nausea and vomiting and reports their legs are tingling. The nurse will review which of the following labs to confirm tumor lysis syndrome? SELECT ALL THAT APPLY A. WBC B. Uric acid C. Potassium D. Platelet count E. Calcium

B. C. E. (rational: Tumor lysis syndrome manifests 1-2 days after chemotherapy and can last 5-7 days. It is a result of massive cell destruction and the intracellular components of the tumor will rapidly release. We will see hyperuricemia, hyperphosphatemia (not listed above), hyperkalemia, and hypocalcemia. Treatment will include prevention of renal failure with fluids and medications to rid the body of uric acid.)

the nurse is providing education to a patient with suspected TB. which of the following should the nurse include when asked about the possible signs and symptoms? SATA A. cough for a min of 6 weeks B. night sweats C. weight gain D. hemoptysis E. Chills

B. D. E. (rational: It is possible to have TB without a cough for 6 weeks. Weight loss often corresponds with TB.)

a nurse is preparing to perform tracheal suctioning on a patient post laryngectomy. what actions will the nurse plan to do for this procedure? SATA A. turn on the suction device and set the regulator to 240mmHg B. assess the heart rate and rhythm and bedside oxygen saturation level C. apply continuous suction and slowly withdraw the catheter while rotating it back and forth D. place the patient in a semi-fowlers position E. observe the patient's color while suctioning

B. D. E. (rational: Vital sign monitoring is important during suctioning since dysrhythmias and hypoxia can occur. Increasing the head of bed allows for better cough and gas exchange. The patient's color can be a sign of their oxygenation status. The regulator must not go above 120 mmHg for suctioning and the nurse needs to limit the time of the pass to 10 seconds and should encircle the trach while withdrawing the catheter instead of having it go back and forth.)

The nurse is completing a home visit for an elderly patient who developed hypernatremia. Which instructions should the nurse include in the plan of care? A. Weigh yourself every morning and every night B. Check your pulse and rhythm every morning and every night C. Read the labels of all packaged foods to determine the sodium content D. When you prepare meals, try to bake or grill food rather than frying it

C. (Rational: Prepackaged foods have a high sodium content. Many clients do not know how to determine the sodium content of such food. Teaching the client how to read labels and calculate the sodium content of food can help him/her adhere to the prescribed sodium restriction and prevent hypernatremia. Option A is incorrect because daily weights, not twice daily weights, are effective in monitoring fluid balance but not for preventing hypernatremia. Option B, checking the pulse is also not related to preventing future episodes of hypernatremia. Option D is incorrect because while grilling is a healthier option the cooking method does not affect the sodium content of a meal. When limiting sodium intake, the patient should avoid adding additional salt and using seasonings without salt)

A healthcare provider is refusing to provide additional pain medication for 25 year old male African American patient who has chronic back pain and continues to have a pain rating of 8 out of 10. The physician feels the patient is just seeking drugs for recreational use. What is the nurse's best action? a. Use additional pain relief measures, such as guided imagery and distraction. b. Administer the medication as ordered. c. Ask the physician to consult with the pain control specialist to best meet this client's needs for pain relief. d. Explain to the client that the lower doses are better for him in the long run so that he will not become addicted again.

C. (Rationale: Many health care providers have biases regarding drug use in racial or ethnic minorities and are not familiar with pain management needs. As nurses we need to believe the patients and do what we can to achieve their goals. Pain management healthcare providers are experienced in this area and should be consulted when pain control cannot be accomplished.)

The nurse is taking a patient's blood pressure manually. When the cuff is inflated, the nurse notes this assessment finding: Which electrolyte imbalance could be the cause of this spasm? A. Hypercalcemia B. Hypokalemia C. Hypocalcemia D. Hypernatremia

C. (Rational: A low calcium (hypocalcemia) causes a positive trousseaus sign, which indicates an early sign of tetany and hyperreflexia. Chvostek's sign is the twitching of the facial muscles in response to tapping over the area of the facial nerve)

The nurse administers albumin IV to a patient with liver failure. Which assessment should the nurse complete first? A. measure abdominal girth B. Weight the patient C. Take blood pressure D. Check urine output

C. (Rational: Albumin will draw fluid from thenintersitital space to the intravascular space. It is considered a volume expander because the fluid will remain in the intravascular space. Therefore, you would want to obtain the BP first. Additional volume will increase BP. Urine output may have increased after administration and as a result the patient's weight may improve; however, this is not immediate.)

The nurse is evaluating the patient after administering ADH for treatment of diabetes insipidus following a brain injury. Which finding indicates that the medication was effective? A. Increased urine output, increased urine osmolarity B. Increased urine output, decreased urine osmolarity C. Decreased urine output, increased urine osmolarity D. Decreased urine output, decreased urine osmolarity

C. (Rational: Antidiuretic hormone binds to receptors in the kidney, increasing only the reabsorption of water, and increasing plasma volume while decreasing urine volume. Sodium reabsorption is not increased by the presence of ADH, so more sodium remains in the urine, increasing its osmolarity. The outcome of ADH influence in decreased output of urine with a higher concentration (osmolarity) )

The patient's serum sodium level is 128mEq/L and serum potassium level is 2.8mEq/L. Which hormonal problem is most likely to have caused this clinical situation? A. increased aldosterone secretion B. Decreased aldosterone secretion C. Increased ADH secretion D. Decreased ADH secretion

C. (Rational: Both increased ADH and aldosterone secretion could result in the low sodium level. Increased aldosterone secretion leads to excessive loss of sodium and water while increasing potassium reabsorption. If decreased aldosterone levels were responsible, the low serum sodium level would be accompanied by a normal or higher than normal level of serum potassium. ADH secretion causes increased reabsorption of water only. Its excessive secretion causes dilutional effects of all serum electrolytes, which is consistent with the client's laboratory values.)

The student nurse notes that dextrose 5% in water (D5W) has an osmolarity of 272 mOsm/L and asks why it is considered a hypotonic solution. Which explanation by the nurse is correct? A. An isotonic solution must have an osmolarity greater than 300 mOsm/L B. Glucose is not an electrolyte and does not completely dissociate in water C. Glucose is immediately metabolized upon administration, leaving pure water in circulation D. IV glucose immediately draws water from the interstitial and intracellular spaces, resulting in hemodilution

C. (Rational: Dextrose 5% in water is isotonic in its container but, upon reaching the blood, glucose is taken up by cells for metabolism (in the nondiabetic client) as rapidly as its infused, with the remainder of the infused fluid being water)

The nurse receives arterial blood gas results of pH 7.0, PaCO2 35, and HCO3 30. Which interpretation of patient's acid-base imbalance is correct? a. Partially compensated metabolic alkalosis b. Uncompensated respiratory alkalosis c. Uncompensated metabolic alkalosis d. Partially compensated respiratory alkalosis

C. (Rational: First, the condition should be recognized as alkalotic due to the high pH. The respiratory value (PaCO2) is within normal range while the bicarb level is elevated leading to metabolic conditions. At this point, you should be able to eliminate b and d. So is it partially compensated or uncompensated - the nurse must refer to the opposite body system. Is PaCO2 on the move? It is still within the normal range so there has been no attempt at compensation leading the nurse to Option C.)

The nurse is caring for a patient who is 1-day post-thyroidectomy. Which assessment findings should prompt the nurse to recheck a serum calcium level? A. Fatigue, hypotension, tetany B. Weakness, tachycardia, disorientation C. Muscle cramps, paresthesia, Chvostek's sign D. Weakness, edema, orthostatic hypotension

C. (Rational: Hypocalcemia is common after a thyroidectomy as the parathyroid glands are also removed. These glands produce the hormone PTH which regulates calcium in the blood and it is responsible)

The nurse is assessing a patient who is currently taking 40mg PO furosemide (Lasix) daily. Which statement made by the patient should the nurse report to the HCP immediately? A. The swelling in my feet has improved B. Sometimes I get a dry mouth C. My legs have been weak and cramping D. I have been urinating more frequently

C. (Rational: Lasix is a loop diuretic that causes fluid, sodium, chloride, and potassium to be excreted via the urine. This medication commonly causes hypokalemia. Option C is correct because cramping and weakness. Option A is incorrect because while this can be a finding with diuresis, edema should improve, and this would be an expect4ed finding. Option B is incorrect because while this can be a finding with diuretic medications, it is a non-specific finding for fluid volume deficit or electrolyte imbalance. Option D is incorrect because with the administration of furosemide, the nurse would expect the patient to have an increased urine output as fluid is removed via the kidneys)

The nurse is providing discharge teaching to a patient with a potassium dietary restriction. Which patient statement demonstrates understanding of this restriction? A. Fruit is very high in potassium and I will need to avoid it B. If I cook the fruit before eating it, the total amount of both potassium and sodium in it will be reduced C. Some fruit is high in potassium, but berries, cherries, apples, and peaches are low in potassium and I can eat them D. Some vegetables, such as potatoes and avocadoes, can be substituted for fruit, so I can still enjoy the texture of raw food

C. (Rational: Not all fruit is potassium rich. Fruits high in potassium include bananas, kiwi, cantaloupe, oranges, and dried fruit. Fruits that are relatively low in potassium and can be included in the diet even when clients much restrict potassium intake include apples, apricots, berries, cherries, grapefruit, peaches, and pineapple)

A patient is admitted with fluid volume excess and is receiving turosemide (Demadex) IV every 12 hrs. Serum potassium levels are being monitored daily: Day 1: 5.6 Day 2: 4.4 Day 3: 3.5 Day 4: 3.1 Which day is best for the nurse to consult with the provider regarding starting potassium replacement? A. Day 1 B. Day 2 C. Day 3 D. Day 4

C. (Rational: The nurse should consult the provider on Day 3 of diuretic therapy. At this point, the patient's level is at the low end of normal and with trending it will continue to decrease as long as the turosemide (Demadex) is being given. Supplementing at this point will prevent further complications related to potassium loss. Day 1 and Day 2, the patient's potassium level is normal and would not need to be supplemented. Day 4, the patient's serum potassium is already low and could lead to further cardiac complications)

The nurse is caring for a patient newly admitted related to a deep vein thrombosis that is being started on a low-molecular weight heparin. What should the nurse include in their patient education? a. The need to avoid foods high in vitamin K b. The need to have blood drawn to measure the INR c. The signs and symptoms of bleeding d. Why this is the preferred medication for anyone with renal failure

C. (Rationale - Heparin along with other anticoagulation medications increase the risk of bleeding. Keeping vitamin K in the diet consistent is important for patients on warfarin. INR is the test used to evaluate and make dose changes to warfarin. Unfractionated heparin is preferred for renal failure patients because of the shorter half-life.)

The nurse is caring for a patient admitted related to a deep vein thrombosis who also has significant edema and pain in the left calf. What should the nurse include with their teaching? a. Use the compression stockings on the legs to promote better blood flow b. Ensure that the legs are dependent to promote better blood flow c. Elevate the leg to decrease the swelling d. The need to increase their intake of protein to recover

C. (Rationale- DVTs are venous issues and elevating the leg will decrease swelling. Compression stockings are contraindicated because it can lead to the clot mobilizing. Keeping the leg dependent allows for better arterial blood flow. There are no dietary implications for recovering from a DVT.)

The nurse is teaching foot care to a patient with peripheral arterial disease. Which statement made by the patient indicates teaching has been effective? a. "Knee high stockings will help improve blood flow to my legs" b. "I can go barefoot in my home" c. "I should avoid crossing my legs" d. "I can change my socks every other day"

C. (Rationale- Knee high stockings are used to promote circulatory return in venous insufficiency. Arterial disease can present with paresthesia, for this reason it is inappropriate to not have protective footwear on the patient. Sock changes should be encouraged daily to promote foot inspection. Patients should avoid crossing their legs to prevent interruption in arterial flow.)

The nurse is caring for a patient with peripheral vascular disease that reports intermittent claudication. What should the nurse include with their teaching? a. Keep the legs elevated above the level of the heart while in bed b. Alternating between hot and cool compresses is likely to help alleviate the discomfort c. It is important to remain active despite experiencing intermittent claudication d. Non-steroidal anti-inflammatory medications like ibuprofen can help with pain relief

C. (Rationale- Peripheral arterial disease is caused by plaque formation. A sedentary lifestyle is likely to contribute to further plaque formation. Elevating legs is for venous issues, arterial issues need extremity dependence to promote blood flow. Compresses and NSAID use would not be appropriate treatments for a patient experiencing claudication pain.)

The nurse is treating a client with metabolic acidosis related to sudden cardiac arrest. Which assessment indicates that treatment has been effective? a. Development of U-wave b. Heart rate 62 bpm c. T waves follow QRS complex at height of 2 mm d. P wave before the QRS complex with PR interval of 0.46 sec

C. (Rationale: Acidosis is typically accompanied by hyperkalemia due to the body's buffer system and the quick exchange for potassium and hydrogen ions. So the treatment for acidosis often includes management of hyperkalemia. A hallmark finding in hyperkalemia is EKG changes which include prolonged PR intervals, wide QRS complexes and tall, peaked T-waves. When evaluating the effectiveness of a treatment, the nurse should look for normal findings on the EKG, indicating that the hyperkalemia has resolved and there are no acute EKG changes. Option C indicates that T waves are present in the correct sequence and at the correct height and would therefore indicate a normal finding. The development of a U-wave is abnormal and would not indicate effective treatment. The heart rate declining to 62 would not indicate that the treatment was effective as bradycardia is not associated with acidosis or hyperkalemia. The P wave should be before the QRS complex but should only have a measurement of 0.2 seconds or less to be normal. A reading of 0.46 seconds would indicate a prolonged PR interval and that the treatment was still ineffective.)

The nurse is completing a triage assessment in the emergency department for a client with shoulder pain. Which question should the nurse include in the interview to best evaluate intensity? A. "What actions have you taken to relieve this pain?" B. "Have any members of your family ever abused drugs?" C. "Is your pain mild, moderate or severe?" D. "How long have you had this pain?

C. (Rationale: Assessing for the intensity of the pain is asking "how bad is the pain" Option C lets the nurse know how severe the pain is and the nurse is able to better make decisions about interventions. Also, baseline information is helpful in determining if interventions were successful.)

An infant is being evaluated in a clinic for a diagnosis of heart failure. Which assessment finding will the nurse anticipate while caring for the patient? A. A recent weight gain of over 3 pounds B. Significant pitting edema in the legs C. Height and weight less than the 25% percentile D. The infant cries much more than other infants

C. (Rationale: Infants with heart failure tend to be small for their age. A 3# weight gain for an adult would be anticipated, that that would be a tremendous amount of weight for an infant. Infants tend to get more periorbital edema instead of edema in the legs. Crying is not a sign of heart failure.)

The nurse is caring for a client following a total hip replacement who has been receiving ketorolac (Toradol) IV every 6 hours for pain. Which assessment finding should the nurse report to the healthcare provider? a. Urine output of 300ml during 8-hour shift b. Creatinine of 1.1 mg/dL c. Weight increase of 2.5 pounds since yesterday d. BUN of 22 mg/dL

C. (Rationale: Ketorolac is an NSAID, an anti-inflammatory drug that is often used in pain management. One of the major side effects of this class of medication is impaired renal function. The nurse should watch for changes related to kidney function, electrolyte management and fluid retention. A weight gain of 2.5 pounds is correct because this is a significant increase and indicates that the client is retaining fluid. Option A, urine output of 300ml during 8-hour shift is incorrect as this is a normal urine output. 30ml/hour or 240ml/8 hours is the minimum urine output for an adult. Option B, creatinine of 1.1 mg/dL is incorrect as this is a normal lab value. The nurse should always compare this to previous data to determine if the patient has had a significant increase. Option D, BUN of 22 mg/dL is incorrect as this value is within normal range. This test is also a measure of renal function but should always be taken into account with creatinine as well as other fluid balance measure such as weight.)

Nitroglycerin and aspirin are prescribed for a patient with angina-like chest pain. Which statement by the nurse best describes the purpose of aspirin? A. "Aspirin will help with the pain without sedation" B. "Aspirin will improve blood flow throughout your body" C. "Aspirin keeps platelet from sticking together and forming a clot " D. "Aspirin improves circulation by increasing the strength of each muscle contraction"

C. (Rationale: Taking aspirin at the first sign of chest pain can disrupt blood clotting and reduce the risk of a coronary thrombosis.)

A client with DKA has a blood sugar of 320 mg/dL, a respiratory rate of 32 breaths/minute and a deep, regular respiratory effort. The nurse should implement interventions related to which acid-base imbalance? a. Respiratory acidosis b. Respiratory alkalosis c. Metabolic acidosis d. Metabolic alkalosis

C. (Rationale: The nurse should implement interventions related to metabolic acidosis. DKA is an acidotic state due to loss of HCO3 and is one of the 3D's of metabolic acidosis. The change in respiratory rate and effort is an attempt by the body to eliminate excess CO2 (Kussmaul respirations). DKA is a metabolic condition whose root cause is not in the respiratory system and Option A and Option B should be eliminated. As the name implied, DKA is an acidotic state so option D should be eliminated.)

A patient who has just started taking sustained-release morphine sulfate (MS Contin) for chronic pain complains of nausea and abdominal fullness. Which action should the nurse take first? a. Consult with the healthcare provider about using a different opioid b. Administer the ordered ondansetron (Zofran) 4 mg IV. c. Listen to bowel sounds and palpate the abdomen d. Instruct the patient to follow a clear liquid diet until the nausea decreases.

C. (Rationale: This is a nursing process question. With the data provided, the nurse should continue to gather assessment data until there is enough data to make a decision. The nurse should begin by completing a focused abdominal assessment and then deciding what the best course of action should be. Option A, B and D are all action steps and could be completed once the nurse have finished gathering relevant data.)

a patient is admitted with lymphoma following chemotherapy. which instruction is most important for the nurse to provide during education? A. monitor protein in urine B. monitor roommate for infection C. wash hands frequently D. visitors should wear a mask when entering the room

C. (rational: A major objective in caring for the patient with leukemia is protection from infection. Frequent handwashing is of the utmost importance. If at all possible, the patient should be in a private room. Masks are worn by anyone having an upper respiratory tract infection. The patient may be on a "minimal bacteria diet." Protein is not a factor in this diet.)

The nurse is assessing the pulse oximetry of a patient. The current reading is 85%, down from 92% about one hour ago. The patient reports no respiratory distress, has a respiratory rate of 18, has pink, warm skin and is resting comfortably. Which action should the nurse take? a. Administer oxygen by mask or cannula. b. Document the finding. c. Verify the measurement. d. Notify the physician.

C. (rational: A pulse oximetry reading in this range indicates desaturation and is considered an emergency requiring immediate intervention. However, the nurse should first verify the accuracy of the reading by measuring pulse oximetry at another body site (nose, earlobe, or forehead). If the measurement shows desaturation at the second site, administer oxygen and notify the physician.)

The nurse is teaching the parents of a child with diabetes. Which agent should the nurse teach the parents to administer if their child suffers a severe hypoglycemic reaction? a. Intravenous (IV) dextrose b. Orange juice c. Subcutaneous glucagon administration d. Subcutaneous insulin administration

C. (rational: Glucagon increases blood glucose levels by stimulating the hepatic production of glucose from glycogen stores (glycogenesis). IV dextrose will further lower the blood glucose level as well as insulin administration. Orange juice would be appropriate for mild hypoglycemia.)

The nurse is infusing insulin intravenously to a patient with extreme hyperglycemia. Which electrolyte abnormality indicates the infusion is too rapid? a. Serum calcium level of 8.0 mg/dL b. Serum chloride level of 90 mEq/L c. Serum potassium level of 3.0 mEq/L d. Serum glucose level of 220 mg/dL

C. (rational: IV insulin is a hypertonic solution and will cause potassium to diffuse into the extracellular space in exchange for insulin moving intracellularly. This shift in electrolytes can occur rapidly if administration is performed too quickly causing transient hypokalemia. Once the blood glucose level is stabilized the potassium will diffuse back into the intracellular space and normalize.)

the nurse is caring for a patient with leukemia following chemotherapy administration. which task is most appropriate to delegate to an LPN? A. monitor the patients protein intake B. assess the client for infection C. assist client with ambulation D. educate the patient on activity restrictions

C. (rational: LPNs cannot assess, teach, or evaluate. The client may be on a "minimal bacteria diet." Protein is not a factor in this diet)

1. A patient has been taking metformin (Glucophage) for six months is scheduled to receive a dose with breakfast. The patients am labs reveal: serum creatinine 1.9 mg/dL; blood urea nitrogen (BUN) 28 mg/dL and serum potassium 5.2 mEq/L. What is the nurse's best action? a. Document the report as the only action b. Instruct the patient to increase his water intake c. Notify the health care provider d. Test a sample of urine for the presence of occult blood

C. (rational: Metformin (Glucophage) is nephrotoxic and based on the renal labs above the patient may be experiencing some renal impairment. It would be most appropriate to notify the physician prior to administering the medication.)

the nurse is teaching a client about a daily dose of INH what should be included in the instructions for the client? A. avoid aspirin and aspirin containing products while taking this drug B. this drug turns your urine red-orange it is a harmless effect C. report any numbness and tingling in your extremities while taking this medication D. you will need to have frequent eye examinations while taking this medication

C. (rational: Numbness and tingling is a side effect of INH. Avoiding aspirin is not necessary while taking the medication. Rifampin is the medication that turns urine red-orange. Ototoxicity is not a known side effect of INH.)

a patient with leukemia is hospitalized for treatment of anemia. what would be a priority action for the nurse? A. provide a diet high in vitamin K B. initiate neutropenic precautions C. alternate periods of rest and activity D. increase fluid intake to 2500mL/day

C. (rational: Nursing care for patients with anemia should alternate periods of rest and activity to maintain patient mobility without causing undue fatigue. High vitamin K diets might be used for a patient with a bleeding disorder. There is no indication that the patient is neutropenic, so isolation is not needed. Increased intake of fluid and fiber will not improve the anemia.)

the nurse is received a prescription to transfuse 1 unit of platelets to a client. which assessment data indicates that the nurse should consult the provided prior to administering? A. patient is fatigued B. blood pressure is 94/56 C. platelets are 42,000 D. blood is oozing from the venipuncture

C. (rational: Platelet transfusions are not usually indicated until the platelet count is below 20,000/ μ l unless the patient is hemorrhaging, so the nurse should clarify the order with the health care provider before giving the transfusion. The other assessment indicate bleeding may be occurring and that the transfusion is appropriate.)

The nurse has instructed a patient diagnosed with COPD how to perform pursed-lip breathing and explains how the technique will assist respirations. Which of the following rationale would the nurse include when teaching the patient about this technique? a. loosen secretions so that they may be coughed up more easily. b. promote maximal inhalation for better oxygenation of the lungs. c. prevent airway collapse and air trapping in the lungs during expiration. d. decrease anxiety by giving the patient control of respiratory patterns.

C. (rational: Pursed-lip breathing increases the airway pressure during the expiratory phase and prevents collapse of the airways, allowing for more complete exhalation. Although loosening of secretions, improving inhalation, and decreasing anxiety are desirable outcomes for the patient with COPD, pursed-lip breathing does not directly impact these.)

The nurse has instructed a patient on the use of their newly prescribed salmeterol (serovent). Which statement indicates that further instruction is needed? a. "I will be certain to shake the inhaler well before I try to take a puff." b. "I will keep using the drug even if I don't notice a difference in my asthma immediately." c. "I will keep the inhaler with me always so that I can get the medicine in my lungs quickly." d. "I will be careful not to let the drug escape out of my nose and mouth."

C. (rational: Salmeterol is designed to prevent an asthma attack; it does not relieve or reverse symptoms. The client does not have to keep this inhaler with him or her always because it is not used as a rescue medication. Salmeterol (Serevent) has a slow onset of action; therefore it should not be used as a rescue drug. The drug must be shaken well because it has a tendency to separate easily. Poor technique on the client's part allows the drug to escape through the nose and mouth.)

The nurse is caring for a diabetic patient. At 1000 he becomes very irritable to the nurse. What is the nurses priority action? a. How well he slept the previous night b. What is upsetting the patient c. When he took his insulin and if he ate his breakfast d. Which behavior on the part of the nurse is upsetting him

C. (rational: Signs and symptoms of hypoglycemia include excessive hunger, confusion, sweating, shakiness, irritability, cool and clammy skin.)

The patient with an asthma attack receives inhaled albuterol/ipratropium via nebulizer. Which assessment finding indicates that further intervention is needed? a. Peak expiratory rate flow 10% below expected value b. Presence of bilateral tactile fremitus c. Suprasternal retraction on inhalation d. Trachea at the midline

C. (rational: The location of the trachea and the presence of tactile fremitus are normal assessment findings. A 10% decrease of peak expiratory rate flow is not significant. Inhalation that causes suprasternal retraction usually means that the client with asthma is having difficulty moving air into the respiratory passages because of airway narrowing and must use accessory muscles. The asthma is not responding to the medication, and the regimen should be changed)

A patient with right lower-lobe pneumonia has been treated with IV antibiotics for 3 days. Which assessment data obtained by the nurse indicates that the treatment is effective? a. The patient coughs up small amounts of green mucus. b. Increased tactile fremitus is palpable over the right chest. c. The patient's white blood cell (WBC) count is 6000/µL. d. Bronchial breath sounds are heard at the right base.

C. (rational: The normal WBC count indicates that the antibiotics have been effective. All the other data suggest that a change in treatment is needed.)

The nurse has instructed a patient on their prescribed long-acting beta 2 agonist. Which statement indicates that the patient understands teaching about the use of this medication? a. "I will not have to take this medication every day." b. "I will take an extra dose of this medication when I have an asthma attack." c. "I will take this medication daily to prevent an acute attack." d. "I will eventually be able to stop using this medication."

C. (rational: This medication will help prevent an acute asthma attack because it is long acting. The client will take this medication every day for best effect. This is not the medication the client will use during an acute asthma attack because it does not have an immediate onset of action. The client will not be weaned off this medication because this is likely to be one of his daily medications.)

The nurse has instructed a patient on the use of a newly prescribed Dry Powder Inhaler (DPI). Which statement made by a patient indicates the need for further instruction? a. "I will not exhale into the inhaler." b. "I will keep the inhaler in the drawer of my bedroom dresser." c. "I will wash the inhaler mouthpiece daily with soap and water." d. "I will inhale twice as hard through this inhaler as I do with my aerosol inhaler."

C. (rational: Washing the DPI may cause the medication in the inhaler to clump together. This action reduces the precision of the delivery of the drug to the client.)

the nurse is preparing to administer the morning dose of pancrelipase (Creon) to a patient diagnosed with CF. which assessment finding would warrant considcering holding the medication? A. the patient reports that their sputum is productive and yellow B. the patient reports that they have taken their insulin 15 minutes ago C. the patient reports not having a bowel movement for three days D. the patient has a procedure scheduled for the next day

C. (rational: constipation is a possible side effect of the medication and the dosage may be decreased. The dose may be titrated based on stool appearance and frequency. There is no interaction with subcutaneous injections and no need to hold the medication before surgery.)

Place the steps for obtaining a peak expiratory flow rate in the order in which they should occur. Display Order a. Take as deep a breath as possible. b. Stand up. c. Repeat the process two additional times, and record the highest number in your chart. d. Make sure the device reads zero or is at base level. e. Blow out as hard and as fast as possible for 1 to 2 seconds. f. Write down the value obtained. g. Place the meter in your mouth, and close your lips around the mouthpiece.

Correct order: a. Make sure the device reads zero or is at base level. b. Stand up. c. Take as deep a breath as possible. d. Place the meter in your mouth, and close your lips around the mouthpiece. e. Blow out as hard and as fast as possible for 1 to 2 seconds. f. Write down the value obtained. g. Repeat the process two additional times, and record the highest number in your chart.

a nurse is caring for a patient post total laryngectomy. the nurse suctions the patient via their tracheostomy. which assessment finding indicated that suction has been effective? A. the patient appears comfortable during the procedure B. no coughing was noted upon initiation of the catheter through the tracheostomy C. telemetry reveals that the patient remains in NSR D. the lungs are fee of rhonchi in the upper lobes

D. (rational: Breath sounds that include rhonchi are heard when the patient has respiratory congestion. Comfort just shows that the patient tolerated the procedure. A lack of a cough while inserting the catheter would be a concern. Remaining in normal sinus rhythm illustrates a lack of a complication rather than having an effective treatment.)

A type 1 diabetic presents to the emergency department with complaints of diarrhea. His ABG's are pH 7.3, PaCO2 35, HCO3 19. Which of the following data is most concerning to the nurse? a. A blood sugar of 140mg/dl b. Potassium of 3.1 mEq/L c. Negative for Trousseau's sign d. Respirations of 24/minute

D. (rational: Kussmaul's respirations accompany diabetic ketoacidosis and are characterized by deep labored breathing, which supports the ABG's. Although the potassium is concerning, the order of ABC's, respiratory will trump a potential circulatory/cardiac issue.)

A patient with CHF has been receiving IV furosemide for 3 days. The occurrence of which assessment findings should indicate to the nurse this medication should be held? A. urine output of 150ml/hr during previous shift B. Heart rate of 52 C. Serum creatinine decreased from 2.1 to 1.6 D. Dizziness with sitting at the bedside

D. (Rational: Furosemide is a loop diuretic that causes fluid, sodium and potassium loss via the urine. It is often used in the diuresis of patients with CHF to decrease circulating blood volume. Dizziness is associated with hypotension and decreased perfusion to the brain and could indicate that the client has lost too much fluid volume. The other options are associated with fluid volume overload)

The nurse is educating a patient with hypophosphatemia. Which statement by the patient indicates the need for further teaching? A. I will eat ore eggs, fish, and meat and avoid dairy products B. I will use Tylenol for pain instead of aspirin C. In the summer, I will drink more water D. If I have indigestion, I will take Tums

D. (Rational: Indigestion of calcium-based products, such as Tums, contributes to hypophosphatemia because calcium and phosphorus are maintained in a balanced, reciprocal concentration in the extracellular fluid)

A patient is admitted to the ED reports feeling weak and having "almost passed out." The patient was gardening outside in temperatures of 100 degrees. The nurse gathers the following assessment findings: Temp: 99.6 Blood pressure: 92/54 Respiratory rate: 24 breaths/min Heart rate: 120 beats/min O2 saturation: 97% RA Neuro: Alert, able to answer questions, dizziness Heart sounds: S1, S2 Lung Sounds: Clear GI: Hypoactive GU: No urine at this time Skin: Dry mucus membranes, tenting turgor Which problem should the nurse identify as the priority? A. Impaired mucus membranes B. High risk for falls C. Decreased cardiac output D. Fluid volume deficit

D. (Rational: The nurse should first identify the abnormal data points in this scenario, including dizziness, almost falling, hypotension, tachycardia, hypoactive bowel sounds and dry mucus membranes with poor skin turgor. once the abnormal data points have been identified, the nurse should organized them using the ABC framework to determine the priority. Circulatory concerns should be the priority and option B should be eliminated first as this is an "at risk" diagnosis and would not take priority over other circulatory concerns. Option A should be eliminated because it does not fall in the priority triangle of heart, brain, lungs, and kidneys. Option C should be eliminated because it the client does not have decreased cardiac output as evidenced by a perfusing systolic pressure of 92 and that they are awake, alert, and talking. The correct response is option D, fluid volume deficit because the patient has all the assessment findings that support this problem (hypotension with tachycardia, dry mucus membranes, dizziness))

A patient has a low parathyroid level (PTH) What effect would this have on serum calcium and phosphate levels? A. High phosphorus and calcium levels B. Low phosphorus and calcium levels C. High calcium and low phosphorus levels D. Low calcium and high phosphorus levels

D. (Rational: When there are low levels of PTH; calcium starts to fall and phosphorus rises. Calcium and phosphorus have an indirect relationship; meaning if one is high the other is low. About 1% of total calcium in the body is found in the cell (outside the bone); although, this calcium found in the blood is critical for communicating information among cells. Calcium stabilizes blood pressure and promotes normal brain function. PTH stimulates the cell wall to allow an influx of calcium in. Phosphorus aids calcium in nerve conduction, muscle contraction and formation of cell membranes. Phosphorus is equally important as calcium for the formation of strong bones and healthy teeth)

Which assessment finding for a patient receiving IV furosemide (Lasix) to treat stage 2 hypertension is most important to report to the health care provider? A. current blood pressure reading of 168/95 B. blood sugar level of 175 C. Orthostatic systolic blood pressure decrease of 12 D. Serum potassium of 3.1

D. (Rationale- hypokalemia is a frequent adverse effect of the loop diuretics and can cause life-threatening dysrhythmias. The health care provider should be notified of the potassium level immediately and administration of potassium supplements initiated. The elevated blood glucose and BP also need collaborative intervention but will not require action as urgently as the hypokalemia. An orthostatic drop of 12 mmHg will require intervention only if the patient is symptomatic.)

A patient arrives in the emergency department with chest pain. Serum enzyme levels indicate an elevated serum creatine kinase (CK-MB) isoenzyme and troponin I. Which action should the nurse take first? A. Prepare the patient for a chest x-ray B. Call SBAR report to admitting RN C. Prepare to administer clopidogrel D. Prepare the patient for transport to the cath lab

D. (Rationale: Cath lab will be the most important intervention at this time.)

The nurse is assessing a client in labor who has had epidural anesthesia for pain relief. Which assessment finding should the nurse identify as a complication of an epidural block? a. Vomiting b. Tachycardia c. Diaphoresis d. Hypotension

D. (Rationale: Epidural anesthesia is when pain relief medication is placed into the epidural space. This direct access to the central nervous system means that healthcare providers can use small amounts of medication to achieve the same effects. Option D, hypotension is correct as hypotension is an adverse effect of an epidural. The nurse should be prepared to administer a fluid bolus if needed. Option A, vomiting is incorrect because it is a typical adverse reaction to oral opioids when given orally. By bypassing the GI tract avoids many of the unwanted GI side effects of opioids. Option B, tachycardia is not an adverse reaction to epidural medications. Option C, diaphoresis, is incorrect as this is an adverse effect of inhaled nitrous oxide. This medication can be given during labor for pain management.)

Which statement by a patient indicates a non-modifiable risk to developing coronary artery disease? A. "I am trying to quit smoking, but it is difficult." B. "I walk at least one mile every day." C. "My blood sugars sometimes run a little high." D. "My father died of a heart attack at age 70."

D. (Rationale: Family history is a non-modifiable risk factor. Smoking, exercise and blood sugar control are modifiable risk-factors.)

The nurse is caring for a client who is taking furosemide (Lasix). Which adverse effect should the nurse monitor for? a. Respiratory acidosis b. Respiratory alkalosis c. Metabolic acidosis d. Metabolic alkalosis

D. (Rationale: Many diuretics, especially loop and thiazide diuretics, increase the excretion of hydrogen ions, leading to excess acid loss through the renal system. This situation causes an acid deficit leading to metabolic alkalosis. Furosemide would not have an effect on the respiratory system related to acid-base balance. Metabolic acidosis occurs when hydrogen ions are retained and furosemide causes hydrogen ion loss via the urine.)

The nurse is preparing to administer morphine to a patient. Which is the priority assessment? a. Blood pressure b. Urine output c. Bowel sounds d. Respiratory rate

D. (Rationale: Morphine is an opioid medication used for pain management that has a list of adverse effects. The nurse should use the ABC framework in this scenario to determine the priority assessment. Option D, respiratory rate, is correct because this would be the priority assessment in the airway, breathing, circulation approach. Morphine can cause respiratory depression and the nurse would consider withholding the medication if the patient was breathing less than 12/breaths/minute. Option A is incorrect because it falls under circulatory concerns which would not take priority over respiratory rate. Option B, urine output, is incorrect because while morphine can cause urinary retention it does not take priority over respiratory concerns. Option C, bowel sounds is incorrect because opioids can cause constipation and hypoactive bowel sounds, this assessment is not a priority over respiratory status.)

The nurse is reviewing the following EKG for a caring for a patient admitted with acute coronary artery syndrome: Which condition does this represent? A. A myocardial infarction in the past B. Mild cardiac ischemia C. Subendocardial injury D. Recent myocardial injury

D. (Rationale: ST segment elevations are caused by myocardial ischemia or infarction.)

A patient reports a history of having substernal chest pain. Which question should the nurse use to determine if the pain is consistent with stable angina? A. "Does your pain always radiate to the jaw?" B. "Does your pain last for at least 15 minutes?" C. "Did your pain occur at rest?" D. "Was your pain precipitated by activity?"

D. (Rationale: Stable angina is chest pain that is precipitated by activity. They are doing an activity that is requiring greater blood flow, and they do not have adequate blood flow. Lactic acid starts to build up and causes pain. We will need to know what the patient was doing when the pain started.)

The nurse is caring for a patient who is sedated and mechanically ventilated. The nurse receives the following ABG report: pH 7.42, PaCO2 40 mm Hg, HCO3 - 26, PaO2 86%. Which intervention should the nurse include in the patient's plan of care? a. Increase oxygen supply by increasing FiO2 b. Decrease respiratory rate on ventilator c. Encourage the client to take deep slow breaths d. Complete repositioning every 2 hours

D. (Rationale: The first step in this question is to interpret the data. You should interpret the ABG as normal with a normal PaO2 as well. Next, ask yourself the question what you would do for a patient who is sedated and ventilated who has a normal ABG. The correct response is to complete repositioning every 2 hours to prevent skin breakdown. Option A would be appropriate if the client had a low PaO2. Option B would be appropriate for respiratory alkalosis in the ventilated patient. Option C would also be appropriate for the patient with respiratory alkalosis who is alert and orientated. As this patient is sedated and ventilated, eliminate this response.)

A patient who has experienced a myocardial infarction develops left ventricular heart failure. Which assessment finding should the nurse report immediately? A. Patient develops a headache B. Blood pressure of 134/70 C. Heart Rate of 60 beats per minute D. Changes in mental status

D. (Rationale: The nurse should remain alert for signs of poor organ perfusion that are the result of poor cardiac output. When the brain is not well perfused, we can start to see changes in mental status. Other signs include urine output less than 30mL/hr, cool, clammy extremities with decreased or absent pulses, fatigue and recurrent chest pain.)

The client has an arterial blood gas pH of 7.48, CO2 47, HCO3 30. Which interpretation of acid-base imbalance is correct? a. Partially compensated Respiratory acidosis. b. Partially compensated Respiratory alkalosis. c. Partially compensated Metabolic acidosis. d. Partially compensated Metabolic alkalosis

D. (Rationale: The pH is high representing an alkalotic state. The bicarb is also elevated which is alkalotic thus creating a metabolic alkalosis. However, with the CO2 increased the body is trying to compensate, since the pH is not back to normal range yet it is only partially compensated.)

A postoperative patient who has undergone extensive bowel surgery moves as little as possible and does not use the incentive spirometer unless specifically reminded. The patient refuses oxycodone, rates the pain severity as an 8 on a 10-point scale but tells the nurse, "I can tough it out." Which explanation by the nurse is best to increase patient compliance? a. "Most patients become addicted to opioids when using them incorrectly.". b. "We can easily manage your constipation with oral medications or enemas." c. "If the oxycodone is not effective, we can try another type of medication." d. "Unrelieved pain can make healing more difficult and lead to respiratory complications."

D. (Rationale: The patient's low activity level, lack of spirometer use, and statement to the nurse indicate that there is a lack of understanding about the purpose of postoperative pain management. The patient did not indicate a concern about becoming addicted, a desire for alternate medications, or anxiety about analgesic side effects.)

The nurse is caring for a client who reports experiencing indigestion with shortness of breath 5 days ago. An elevation in which lab value would indicate that the patient experiences a myocardial infarction? A. CK-MB B. C-reaction protein C. Myoglobin levels D. Troponin I levels

D. (Rationale: Troponin levels elevate after 2 to 6 hours and peak after 12-26 hours. They will return to baseline over 5-14 days. CKMB levels elevate after 4 to 6 hours and peak after 24 hours They will return to baseline over 2-3 days. Troponin is the preferred biomarker because it is a more specific and sensitive marker for heart muscle damage than CK-MB.)

The physician orders scheduled NPH (Novalin) 15 units and lispro (Humalog) 5 units for a patient with type 1 diabetes at 0900. The patient also has sliding scale lispro (Humalog) before meals. This am the patients' blood sugar is 238 mg/dl. How much insulin will the nurse administer for the am shift? Sliding Scale Insulin Humalog R (lispro) 0-149 mg/dl = 0 units 150-199 mg/dl = 2 units 200-249 mg/dl = 4 units 250-299 mg/dl = 6 units 300-349 mg/dl = 8 units 350-399 mg/dl = Call Physician a. 21 units b. 25 units c. 26 units d. 24 units

D. (rational: 15 units of NPH (Novalin) + 5 units of scheduled lispro (Humalog) + 4 units of sliding scale lispro (Humalog = 24 units)

a patient with sickle cell disease was admitted to the hospital with splenomegaly and abdominal pain. which statement should the nurse include is discharge teaching? A. avoid drinking large amounts of water B. eat six small meals daily C. engage in strenuous anaerobic exercise three days a week D. receive a yearly influenza vaccination

D. (rational: Abdominal pain and a palpable spleen could indicate blood trapping in the spleen. Over time, the spleen may become nonfunctional, which makes the patient at risk for bacterial infection. An annual influenza vaccination helps prevent infection. A patient with sickle cell disease should not become dehydrated or engage in strenuous physical activity because this could precipitate a crisis. Eating smaller meals has no impact on sickle cell disease.)

the nurse is caring for a patient recovering from a lung resection surgery for cancer. the patient asks the nurse about the purpose of the chest tube after surgery. which response by the nurse would be best? A. the chest tube allows patients to deep breath and cough more easily to prevent pneumonia B. it is placed in the pericardial area to drain fluid is a result of surgery C. it is used until with can keep your oxygen levels at an appropriate level without the use of supplemental oxygen D. it is used to allow the lung to re-expand after surgery

D. (rational: After lung resection surgeries, air must be removed to allow the lung to expand. Chest tubes often are uncomfortable and will hinder cough efforts. The tube is placed in the pleural instead of pericardial space. Oxygen is not a clinical indicator that the chest tube can be removed. Instead, the lack of an air leak and minimal chest tube output are the indicators that it is able to be removed.)

The nurse is caring for a patient in status asthmaticus. Which assessment data would indicate the need for intubation and mechanical ventilation? a. ventricular dysrhythmias and dyspnea occur. b. loud wheezes are audible throughout the lungs. c. pulsus paradoxus is greater than 40 mm Hg. d. fatigue and an O2 saturation of 88% develop.

D. (rational: Although all of the assessment data indicate the need for rapid intervention, the fatigue and hypoxia indicate that the patient is no longer able to maintain an adequate respiratory effort and needs mechanical ventilation. The initial treatment for the other clinical manifestations would initially be administration of rapidly acting bronchodilators and oxygen.)

The nurse assesses the chest of a patient with pneumococcal pneumonia. Which finding would the nurse expect? a. Hyperresonance to percussion b. A grating sound on auscultation c. Dry, nonproductive cough d. Increased tactile fremitus

D. (rational: Increased tactile fremitus over the area of pulmonary consolidation is expected with bacterial pneumonias. Dullness to percussion would be expected. Pneumococcal pneumonia typically presents with a loose, productive cough. Adventitious breath sounds such as crackles and wheezes are typical. A grating sound is more representative of a pleural friction rub rather than pneumonia.)

A patient asks the nurse why lispro is given immediately before meals. What is the nurses most appropriate response? a. All insulins are given prior to meals for maximum effectiveness b. Lispro is better absorbed on an empty stomach c. Lispro requires multiple injections per day d. There is a risk of low blood sugar if lispro is given too early before meals

D. (rational: Lispro is a rapid acting insulin. The onset for rapid acting insulin 15-30 minutes, in which food needs to be consumed within the timeframe of the onset or the patient may experience an adverse event.)

A patient is receiving metformin (Glucophage) for type 2 diabetes. The patient is scheduled for a CT scan with contrast dye injection. Which action made by the nurse is most appropriate? a. Given as scheduled before and after the test b. Given the day before the test, and will be held the day after the test c. Held the day before the test, but will be given immediately after the test d. Held the day before the test, the day of the test, and the day after the test

D. (rational: Metformin along with contrast dye are nephrotoxic and can cause moderate to severe renal impairment.)

a 14-year-old girl newly diagnosed with iron deficiency anemia is preparing for discharge. which of the following activities best describes the nurse's role as a patient advocate? A. arranging for a visit with a home health nurse B. providing written medication instructions to the patient's parents C. instructing the patient to follow up with her primary provider in 4 weeks D. teaching the patient dietary choices that are rich in iron

D. (rational: Teaching the patient how to appropriately choose dietary sources high in iron is an action that directly helps the patient develop independence in disease managent. While A, B, and C may be helpful directions in this patient's plan of care, they do not provide the patient with the necessary skills to manage her anemia independently.)

the nurse is caring for a child admitted to the hospital for treatment of cystic fibrosis. which finding would be consistent with this diagnosis? A. decrease heart rate and increase blood pressure B. frequent bloody stools and oral lesions C. increased body weight with and increased proportion of body fat D. thick and sticky mucus

D. (rational: Thick mucous is the result of loss of sodium from CF. Vital sign changes and bloody stools are not characteristics of CF. Weight loss and lack of body fat are also often seen with CF.)

the nurse is planning care for a patient that underwent radial neck dissection and laryngectomy for cancer. the patient is tearful when they wake up from surgery. which response by the nurse is best? A. I can see that you are upset so Ill do my best to get your family in the room quickly B. Tell me what is going on. talking through your problems can be very therapeutic C. our hospital has wonderful support services that are available to help you during this difficult time D. I can see that this is diffiult for you I am here to help you

D. (rational: making an observation is a therapeutic technique. Getting the family into the room is jumping to conclusions. The patient is unable to talk after surgery because the larynx has been removed. Making a referral without first addressing the issue with the patient is unnecessary.)

A nurse is caring for a client who is using a patient-controlled analgesia (PCA) pump for postoperative pain management. The nurse enters the room to find the client asleep and his partner pressing the button to dispense another dose. Which is the best response by the nurse? a. "Next time you think he needs more medication, call me and I'll push the button." b. "It's a good idea to help make sure your husband can sleep comfortably." c. "Why do you think your husband needs more medication when he is asleep?" d. "Your husband should decide when more medication is needed."

D. (Rationale: A PCA has many benefits for patients including timely pain management and a sense of control over their pain. Effective teaching is essential to the success of this intervention. The client should be taught how to use the button, to press it when they feel the pain beginning rather than when it is at its worst, how often they can push it and what type of medication they are receiving. Patient's family or caregivers should be taught to monitor for common side effects, such as depressed breathing and drowsiness. An important teaching point to avoid overdose is that only the patient should push the button. Option D, "Your husband should decide when more medication is needed, is correct as the nurse should explain to the client's partner that the client is the only one who should operate the PCA pump. In situations where the client is not able to do so, the provider may authorize a nurse or a family member to operate the pump. Option A, "Next time you think he needs more medication, call me and I'll push the button." Is incorrect as the nurse should administer a PRN or around-the-clock dosing if the client is having breakthrough pain, but should not push the client's PCA button. Option B, "It's a good idea to help make sure your husband can sleep comfortably." Is incorrect because the nurse should determine with via client assessment if there is breakthrough pain that may require more pain medication. The best pain assessment measure is subjective and should be gathered via interview. Option C, "Why do you think your husband needs more medication when he is asleep?" is incorrect because the nurse's goal is to educate the client's partner. Asking "why" questions can make the partner defensive. This is not gathering assessment data either, as the nurse already has enough information to make a decision in this situation.)


संबंधित स्टडी सेट्स

Types of Understatement and Irony (Figures of speech)

View Set

NURS 482 - Final Exam Practice Questions (Module 5-7)

View Set

Chapter 17.3 and 17.5 Connect Conceptual, Comprehensive, and Concept Questions

View Set

Final COMLEX/USMLE STEP 1 Study Set

View Set

18FALL-SOC1051-001 - Introductory Sociology (ch. 11/12 homework/review)

View Set